You are on page 1of 51

C H A P T E R

9
Ray Optics and
Optical Instruments
A Quick Recapitulation of the Chapter
1. Laws of Reflection (ii) The ratio of the sine of angle of incidence (i) to the
(i) The incident ray, reflected ray and the normal to sine of angle of refraction (r) is constant, i.e.,
the reflecting surface at the point of incidence sin i
= n21
lie in the same plane. sin r
(ii) The angle of incidence, ∠i = angle of reflection, where, n21 is a constant, called the refractive index
∠r. of the second medium with respect to the first
2. The focal length of a spherical mirror is half of its medium.
R This is known as Snell’s law of refraction.
radius of curvature, i.e., f = .
2 7. A ray of light travelling from denser medium to a rarer
3. Mirror Equation If an object is placed at a medium is incident at the interface of two media at an
distance u from the pole of a mirror and its image is angle greater than the critical angle for the two media,
formed at a distance v from a pole. Then, the ray is totally reflected back to denser medium, this
according to the mirror equation given by phenomenon is called Total Internal Reflection (TIR).
1 1 1
+ = n
v u f 8. sinc = 1
n2
4. If a thin object of linear height h is placed
perpendicular to the principal axis of a mirror and where, c is the critical angle.
the linear height of its image be h′, then linear 9. Refraction at a Spherical Surface If an object is
magnification is given by placed in a medium of refractive index n, at a distance
h′ v u from the pole of a spherical surface of radius of
m= =−
h u curvature R and after refraction, its image is formed in a
5. Refraction of light is the phenomenon of bending medium of refractive index n2 at a distance, v, then
of a ray of light while passing from one transparent n2 n1 n2 − n1
− =
medium to another. v u R
6. Laws of Refraction This equation holds for any curved spherical surface.
(i) The incident ray, the refracted ray and the 10. Lens Maker’s Formula is given by
normal to the interface at the point of incidence,
all lie in the same plane. 1 1 1
= (n21 − 1)  − 
f  R1 R 2 
11. The lens formula is given by 17. The phenomenon of splitting of light into its
1 1 1 component colours is known as dispersion.
− =
v u f 18. Magnifying power of simple microscope,
12. Magnification (m ) produced by lens is defined as the D
m =1+
ratio of the size of the image to that of the object. f
h′ v where, D is the least distant of distinct vision.
m= =
h u 19. Angular magnification of simple microscope is
13. Power of a lens is a measure of the convergence or D θi
divergence which a lens introduces in the light falling on it. m= =
f θo
1 100
P = = dioptre
f (m) f (cm) 20. Magnifying power of a compound microscope is
It is also defined as the tangent of the angle by which it L  D
m = mo me = 1 + 
converges or diverges a beam of light falling at unit fo  fe 
distant from the optical centre.
where, fo = the focal length of objective lens
tan δ = h / f and fe = the focal length of eyepiece.
14. If several thin lenses of focal length f1, f2 , f3 , … are in
21. Magnifying power of an astronomical telescope is
contact, the effective focal length of their combination
is given by β fo
m= =
1 1 1 1 α fe
= + + , … or P = P1 + P2 + P3, …
f f1 f2 f3 where, β = angle subtended at the eye by the final
15. Angle of deviation through a prism, δm = i + e − A image
where, i is the angle of incidence, e is the angle of α = angle which the object subtends at the
emergence and A is the angle of prism. lens or the eye.
 A + δm 
sin   22. Length of the telescope tube = fo + fe
 2 
16. Refractive index of prism material, n =
sin ( A / 2)
where, δm is minimum deviation.

Objective Questions Based on NCERT Text

Topic 1
Reflection of Light by Spherical Mirrors
1. Human eye can sense (c) few rays moving in different directions
(a) a small range of electromagnetic rays (d) more than one ray
(b) a large range of electromagnetic rays 4. Earlier, we have studied that light is an
(c) cannot detect electromagnetic waves
electromagnetic wave but from our interactive
(d) only longitudinal waves
notion, we realise that light travels in a straight line.
2. Speed of light in vacuum is Which of the given point justify both the
(a) highest attainable speed contradictory facts?
(b) attainable by small mass particles in vacuum (a) Of the order of wavelength of light
(c) attainable by dust particles in interstellar space (b) Much larger than wavelength of light
(d) None of the above (c) Of the order of one millimetre
3. A beam of light is (d) The wavelength of light is much smaller compared to
(a) a ray of light the size of ordinary objects
(b) a bundle of rays
5. If θ 1 and θ 2 are the angles of incidence and reflection 12. Two plane mirrors are inclined to each other such that
respectively, then correct relation between them is a ray of light incident on the first mirror and parallel to
π the second is reflected from the second mirror parallel
(a) θ1 = + θ2 (b) θ1 > θ 2 to the first mirror. The angle between the two mirrors is
2
(a) 30° (b) 45° (c) 60° (d) 75°
(c) θ1 < θ 2 (d) θ1 = θ 2
13. Two plane mirrors are inclined at an angle θ. It is
6. Incident ray always lies in a plane found that a ray incident on one mirror at any angle is
(a) perpendicular to plane containing normal and reflected rendered parallel to itself after reflection from both
ray the mirrors. The value of θ is
(b) perpendicular to normal
(a) 30° (b) 60°
(c) perpendicular to plane containing reflected ray
(c) 90° (d) 120°
(d) containing reflected ray and normal
14. Four identical mirrors are made to B C
7. Laws of reflection are valid in case of stand vertically to form a square
(a) regular reflection arrangement as shown in a figure. A
(b) irregular reflection ray starts from the mid-point M of
(c) reflection over a curved surface mirror AD and after two reflections θ
(d) Both (a) and (c) reaches corner D. Then, angle θ A D
M
8. A plane mirror is placed along the X -axis facing must be
negative Y -axis. The mirror is fixed. A point object is (a) tan −1 ( 0.75 ) (b) cot −1 ( 0.75 )
moving with 3 $i + 4 $j infront of the plane mirror. The (c) sin −1 ( 0.75 ) (d) cos −1 ( 0.75 )
relative velocity of image with respect to its object is
Y 15. In given diagrams, point F is

X
P
C F

(a) − 8$j (b) 8 $j


(I)
(c) 3 $i − 4 $j (d) −6 $j

9. A ray of light gets reflected from the plane mirror. The


incident ray and the reflected ray are perpendicular to
each other. The angle of incidence is P F C
(a) 60° (b) 30° (c) 45° (d) 0°
10. A ray of light is incident on a plane mirror along the (II)
direction given by vector, A = 2$i − 3$j + 4k.
$ Find the
unit vector along the reflected ray. (Take, normal to
the mirror along the direction of vector,
B = 3$i − 6$j + 2k).
$ C F
P
− 94 i$ + 237$j + 68k$ − 94 i$ + 68$j − 273 k$
(a) (b)
49 29 49 29 Focal plane
3i$ + 6$j − 2k$ (III)
(c) (d) None of these
7 (a) focal in I, centre of curvature in II and focus in III
1 (b) focal in I and II and centre of curvature in III
11. A ray of light travelling in the direction ( $i + 3 $j ) is
2 (c) focal in I and centre of curvature in II and III
incident on a plane mirror. After reflection, it travels (d) focal in all I, II and III
1
along the direction ( $i − 3 $j ). The angle of 16. In reflection over a spherical mirror, ray parallel to
2 principal axis, after reflection from mirror pass through
incidence is [JEE Advanced 2013] (a) focus (b) centre of curvature
(a) 30° (b) 45° (c) 60° (d) 75° (c) pole of mirror (d) any point
17. A ray passing through or directed towards centre of 23. A concave mirror of focal length f 1 is placed at a
curvature of a spherical mirror is reflected such that it distance of d from a convex lens of focal length f 2 . A
trace back of its path, because beam of light coming from infinity and falling on this
(a) it does not follow law of reflection convex lens-concave mirror combination returns to
(b) angle of incidence is 0° infinity. The distance d must be equal
(c) centre of curvature is midway between object and pole [CBSE AIPMT 2012]
(d) distance of centre of curvature from focus is equal to its (a) f1 + f2 (b) − f1 + f2
distance from pole
(c) 2 f1 + f2 (d) −2 f1 + f2
18. If lower half of a concave mirror is blackened, then 24. An object 2 cm high is placed at a distance of 16 cm
(a) image distance increases from a concave mirror, which produces a real image
(b) image distance decreases 3 cm high. What is the focal length of the mirror?
(c) image intensity increases
(a) − 9.6 cm (b) − 3.6 cm
(d) image intensity decreases
(c) − 6.3 cm (d) − 8.3 cm
19. An object is placed at 10 cm from a concave mirror of 25. A rod of length 30 cm lies along the principal axis of
radius of curvature 15 cm, then a concave mirror of focal length 10 cm in such a way
(a) it forms an erect image that its end closer to the pole is 20 cm away from the
(b) it forms a small and inverted image mirror. The length of the image is [CBSE AIPMT 2012]
(c) it forms a point image (a) 10 cm (b) 15 cm (c) 2.5 cm (d) 5 cm
(d) if forms a magnified, real and inverted image
26. A concave mirror has a radius of curvature of 20 cm.
20. Rear view mirror of a car is of radius of curvature The image of a object formed in mirror is 2.50 times
R = 2 m. A jogger approaches car (from behind) at a the size of the object. How far is the mirror from the
speed of 5 ms −1 . The speed of image, when jogger is object?
39 m from the mirror, is
(a) 5.5 cm (b) 40 cm (c) 6 cm (d) 10 cm
(a) 0.3 ms −1 (b) 0.1 ms −1
(c) 0.5 ms −1 (d) 0.5 cm s −1 27. A container is filled with water (µ =1.33) upto a
height of 33.25 cm. A concave mirror is placed 15 cm
21. A boy 1.5 m tall with his eye level at 1.38 m stands above the water level and the image of an object
before a mirror fixed on a wall. The minimum length of placed at the bottom is formed 25 cm below the water
mirror required to view the complete image of boy is
(a) 0.75 m (b) 0.06 m (c) 0.69 m (d) 0.12 m
level. The focal length of the mirror is
Mirror
22. A short linear object of length b lies along the axis of
a concave mirror of focal length f at a distance u
from the pole of the mirror. The size of the image is 15 cm
approximate equal to
1/ 2 1/ 2
u − f  f  µ=1.33 25 cm
33.25
(a) b   (b) b   cm
 f  u − f
I
2
u − f  f 
(c) b   (d) b   (a) 10 cm (b) 15 cm (c) −18.3 cm (d) 25 cm
 f  u − f

Topic 2
Refraction and Total Internal Reflection
28. Incident ray, normal at the point of incidence and 29. A ray of light strikes an air-glass interface at an angle
refracted ray are always of incidence ( i = 60° ) and gets refracted at an angle of
(a) mutually perpendicular refraction r. On increasing the angle of incidence
(b) inclined at acute angles ( i > 60° ), the angle of refraction r
(c) parallel (a) decreases (b) remains same
(d) coplanar (c) is equal to 60° (d) increases
30. For the same angle of incidence, the angles of 35. For refraction through a plane glass slab, dimension d
refraction in media P , Q and R are 35° , 25° and15°, is called
respectively. Which of the following relation hold true i
for the velocity of light in medium P , Q and R? ε
(a) vP < vQ < vR
(b) vP < vR < vQ ε
(c) vP > vQ > vR e d
(d) vP > vR > vQ (a) refraction shift
(b) lateral shift
31. A divergent beam of light from a point source S
(c) emergence shift
having divergence angle α falls symmetrically on a
(d) incidence shift
glass slab as shown in the figure. The angles of
incidence of the two extreme rays are equal. If the 36. When an object lying in a denser medium is observed
thickness of the glass slab is t and its refractive index from rare medium, then real depth of object is
is n, then the divergence angle of the emergent beam is (a) more than that observed
S (b) less than that observed
α (c) equals to observed depth
i i (d) depends on angle of vision
37. A beaker contains water upto height h1 and kerosene
n t of height h2 above water so that the total height of
(water + kerosene) is ( h1 + h2 ). Refractive index of
water is µ 1 and that of kerosene is µ 2 . The apparent
(a) zero (b) α shift in position of the bottom of the beaker shown
(c) sin −1 (1/ n ) (d) 2 sin −1 (1/ n ) viewed from above is
 1  1
32. A ray of light strikes a material’s slab at an angle of (a) 1 −  h 2 + 1 −  h1
 µ1   µ2
incidence 60°. If the reflected and refracted rays are
perpendicular to each other, then which of the given  1  1
(b) 1 +  h 1 + 1 +  h2
options is are correct?  µ1   µ2
(a) The angle of refraction is 45°  1  1
(b) The angle of refraction is 30° (c) 1 −  h 1 + 1 −  h2
 µ1   µ2
(c) The refractive index of the material is 3
 1  1
(d) Both (b) and (c) (d) 1 +  h 2 − 1 +  h1
 µ 1  µ 2
33. A ray of light strikes a transparent rectangular slab of
refractive index 2 at an angle of incidence of 45°. real depth
38. The ratio is equal to
The angle between the reflected and refracted ray is apparent depth
(a) 75° (b) 90° (a) refractive index of denser medium with respect to air
(c) 105° (d) 120° (b) refractive index of denser medium with respect to rare
medium
34. As you can seen from Fig. (i) and (ii) shows (c) refractive index of rare medium with respect to air
refraction of light from air to glass and from air to (d) refractive index of rare medium with respect to denser
water. Find out the value of the angle θ in the case of medium
refraction as shown in figure (iii) will be
39. A beaker of depth a is half filled with olive oil of
40° 50°
refractive index µ 1 and the other half is filled with
(i) Glass (ii) (iii) θ
Glass
water of refractive index µ 2 . The apparent depth of
Air
Air Water Water the beaker when viewed from above is
50° 45° a(µ 1 + µ 2 ) aµ 1µ 2
45° (a) (b)
2µ 1µ 2 2(µ 1 + µ 2 )
(a) 30° (b) 50° aµ 1µ 2 2a(µ 1 + µ 2 )
(c) (d)
(c) 40° (d) 45° (µ 1 + µ 2 ) µ 1µ 2
40. A vessel is filled with water to a height of 13 cm. The 46. A green light is incident from the water to the
apparent depth of a screw lying at the bottom of the air-water interface at the critical angle (θ). Select the
vessel is measured by a microscope to be 8.5 cm. If correct statement. [JEE Main 2014]
water is replaced by a liquid of refractive index 1.70 (a) The entire spectrum of visible light will come out of the
upto the same height. Find the distance at which water at an angle of 90° to the normal
microscope have to be moved to focus on the screw (b) The spectrum of visible light whose frequency is less
again? than that of green light will come out of the air medium
(a) 0.85 cm (b) 0.52 cm (c) 0.65 cm (d) 1.02 cm (c) The spectrum of visible light whose frequency is more
than that of green light will come out to the air medium
41. Early sunrise and delayed sunset are caused due to (d) The entire spectrum of visible light will come out of the
(a) bending of light rays towards centre of earth due to water at various angles to the normal
gravity
(b) bending of light rays away from centre of earth due to 47. Two transparent media A and B are separated by a
refraction plane boundary. The speed of light in medium A is
(c) bending of light rays towards centre of earth due to 2 × 10 8 ms −1 and in medium B is 2.5 × 10 8 ms −1 . The
refraction critical angle for which a ray of light going from A to B
(d) scattering of light from molecules of gases and dust is totally internally reflected is
particles in atmosphere  1  2
(a) sin −1   (b) sin −1  
42. Time-period of rotation of earth on its own axis is  2  5
24 h. Time taken by sun to shift by 1°, when viewed  4  3
(c) sin −1   (d) sin −1  
from earth is  5  4
(a) 36 min (b) 24 / 360 min
48. A ray of light is incident at the glass-water interface
(c) 4 min (d) around 1 min
at an angle i, it emerges finally parallel to the surface
43. A ray of light strikes an air-glass interface such that a
of water, then the value of µ g would be
part of it is reflected into air and the rest enters glass
Air
as shown in the figure given below.
ay µw = 4/2
dr
f l ecte Water
Incident Re Glass
ray α
Air
Glass (a) ( 4 / 3 ) sin i (b) 1/sin i (c) 4 / 3 (d) 1

Refracted
49. A ray of light from a denser medium
strikes a rarer medium at an angle of
If angle of refraction and refractive index of glass i r
incidence i (see figure). The Denser
with respect to air is r and µ respectively, then value reflected and refracted rays make an Rarer
of α is angle of 90° with each other. The r′
(a) r (b) µ sin r angle of reflection and refraction are
 sin r r and r ′. The critical angle is
(c) sin −1 (µ sin r ) (d) sin −1  
 µ  (a) sin −1 (tan r ) (b) sin −1 (cot i )
44. In total internal reflection, (c) sin −1 (tan r′ ) (d) tan −1 (sin i )
(a) light ray travelling through a denser medium is 50. A right-angled prism is to be made by selecting a
completely reflected back to denser medium proper material and angles A and B ( B ≤ A ), as shown
(b) light ray travelling through a denser medium is in figure. It is desired that a ray of light incident on
completely refracted to rare medium
face AB emerges parallel to the incident direction
(c) light ray is partially reflected back to denser medium
and partially refracted to rare medium
after two internal reflections. What should be the
minimum refractive index n for this to be possible?
(d) light ray is absorbed completely by denser medium
45. Total internal reflection of a light ray travelling from A B
denser medium to rare medium occurs only when
angle of incidence is
(a) 45° (b) 90° 90°
(c) acute (d) more than a certain value
C
1 1 Region I Region II Region III Region IV
(a) n min = (b) n min =
sin A sin B n0 n0 n0
n0
sin A 2 6 8
(c) n min = (d) n min = sin A × sin B
sin B  3  1
(a) sin −1   (b) sin −1  
51. A parallel sides slab ABCD of refractive index 2 is  4  8
sandwiched between two slabs of refractive indices  1  1
(c) sin −1   (d) sin −1  
2 and 3 as shown in the figure. The minimum  4  3
value of angle θ such that the ray PQ suffers total
56. Mirage in hot deserts occurs due to
internal reflection at both the surfaces AB and CD is (a) reflection of light
Q µ2 = √ 2 (b) refraction of light
A B
θ
(c) total internal reflection of light
µ1 = 2 (d) scattering of light
D
µ3 = √ 3
C 57. A diamond piece has more brilliance than a glass
piece of same shape and size, because
(a) 30° (b) 45° (c) 60° (d) 75° (a) diamond has tetrahedral arrangement of carbon atoms
52. A glass prism of refractive index 1.5 is immersed in (b) diamond has more mass density than the glass
water (refractive index 4/3). A light beam incident (c) diamond is more hard than glass
normally on the face AB is totally reflected to reach (d) critical angle for diamond is less than that of glass
the face BC, if 58. Prisms are used in many optical instruments for
B A bending light rays by 90° or 180°. This is possible
because
(a) light undergoes total internal reflection
C (b) light undergoes refraction while passing through a
prism
(c) light shows dispersion while passing through a prism
(a) sin θ > 8/ 9 (b) 2/ 3 sin θ < 8/ 9 (d) prism are easy to mount in corners
(c) sin θ < 2/ 3 (d) None of these 59. An optical fibre is
53. A light source is placed at a depth of d below the (a) a fibre optically visible in light
surface of water (µ). A wooden disc is placed on the (b) a fibre optically invisible in light
surface so that light from the source is not visible from (c) a fibre through which light can travel
the surface. Find out the radius of the wooden disc. (d) a fibre opaque to ordinary light
d 2d
(a) (b) 60. In an optical fibre (shown), correct relation of
(µ − 1)
2 1/ 2
(µ − 1)1 / 2
2
refractive indices of core and cladding is
d
(c) (d) 2d (µ − 1)2 1/ 2
n2
2(µ 2 − 1)1 / 2
Core n1
54. If the critical angle for light going from medium A to B
is θ. Then find the speed of light in medium B, if Cladding
speed of light is v in medium A.
v (a) n1 = n2 (b) n1 > n2
(a) v(1 − cos θ ) (b) (c) n1 < n2 (d) n1 + n2 = 2
cos θ
v
(c) (d) v(1 − sin θ ) 61. An optical fibre is used in
sin θ (a) telescope (b) microscope
55. A light beam is travelling from θ (c) endoscope (d) periscope
region I to region IV (refer
n0
figure). The refractive index in 62. Which of the following is based on the phenomenon
n n n of total internal reflection of light?
regions I, II, II and IV are n0 , 0 , 0 and 0 ,
2 6 8 (a) Sparkling of diamond
respectively. The angle of incidence θ for which the (b) Optical fibre communication
beam just misses entering region IV is (c) Instrument used by doctors for endoscopy
(d) All of the above
Topic 3
Refraction at Spherical Surfaces and by Lenses
63. Consider a convex surface separating two media of 68. A mark placed on the surface of a sphere is viewed
refractive indices n1 and n2 respectively, n2 > n1 . through glass from a position directly opposite as
Which of these is a correct diagram? shown in the figure. The diameter of the sphere is
30 cm and refractive index of glass is 1.5. The
n1 n2 n1 n2 position of the image is
A
(a) (b)
O I O I
View
Mark side
O O B
n1 n2 n1 n2

(c ) (d)
O I O I B
(a) at a distance of 60 cm from surface ABC in the
direction of incident light
(b) at a distance of 60 cm from the surface ABC opposite
64. For the refraction shown below the correct relation is, to the direction of incident light
i (c) at a distance of 30 cm from the surface AOC opposite
n1 n2
r to the direction of incident light on ABC
(d) Both (b) and (c)
O P C I
u R 69. A parallel beam of light is incident on a solid
v transparent sphere of a material of refractive index n.
If a point image is produced at the back of the sphere,
n 2 n1 n 2 − n1 n1 n 2 n 2 − n1 the refractive index of the material of sphere is
(a) − = (b) − =
v u R v u R (a) 2.5 (b) 1.5 (c) 1.25 (d) 2.0
n1 n 2 n1 − n 2 n 2 n1 n1 − n 2
(c) − = (d) − = 70. A point source of light at the surface of a sphere
v u R v u R comes as a parallel beam of light and emerge from the
65. Light from a point source in air falls on a spherical opposite surface of the sphere. The refractive index of
glass surface (n =1.5 and radius of curvature = 20 cm). the material of the sphere is
The distance of the light source from the glass surface (a) 1.5 (b) 5/3 (c) 2 (d) 2.5
is 100 cm. Image distance from the glass surface is 71. A wire mesh consisting of very small squares is
(a) 20 cm (b) 50 cm viewed at a distance of 8 cm through a magnifying
(c) 100 cm (d) 75 cm converging lens of focal length 10 cm, kept close to
the eye. The magnification produced by the lens is
66. A magician during a show makes a glass lens with
n =1.47 disappear in a trough of liquid. (a) 5 (b) 8 (c) 10 (d) 20
Refractive index of the liquid is 72. For a given lens, the magnification was found to be
(a) 1.47 (b) 1.33 twice as large as when the object was 0.15 m distant
4 12 from it as when the distance was 0.2 m. The focal
(c) (d)
3 5 length of the lens is
67. An object is placed in front of a sphere of radius R at (a) 1.5 m (b) 0.20 m
a distance x from the first surface. The value of x for (c) 0.10 m (d) 0.05 m
which the light after refraction from the first surface 73. A diminished image of an object is to be obtained on
becomes parallel to the axis is a screen 1 m from it. This can be achieved by using
R R
(a) (b) (a) plane mirror
(µ − 1) µ +1 (b) a convex mirror of suitable focal length
2R Rµ (c) a convex lens of focal length less than 0.25 m
(c) (d)
(1 +µ ) (µ + 1) (d) a concave lens of suitable focal length
74. A real image of distant object is formed by a 80. A hollow double concave lens is made of very thin
planoconvex lens on its principal axis. Spherical transparent material. It can be filled with air or either
aberration of two liquids L1 or L2 having refractive indices n1
(a) is absent and n2 , respectively ( n1 > n2 > 1). The lens will
(b) is smaller, if the curved surface of the lens faces the object diverge a parallel beam of light if it is filled with
(c) is smaller, if the plane surface of the lens faces the object (a) air and placed in air (b) air and immersed in L 1
(d) is the same whichever side of the lens faces the object (c) L 1 and immersed in L 2 (d) L 2 and immersed in L 1
75. A spherical aberration in a thin lens can be reduced by 81. A thin lens of glass (µ =1.5) of focal length ± 10 cm is
(a) using a monochromatic light immersed in water (µ = 1.33). The new focal length is
(b) using a doublet combination
(a) 20 cm (b) 40 cm (c) 48 cm (d) 12 cm
(c) using a circular annular mark over the lens
(d) increasing the size of the lens 82. Which of the following is true for
76. A double convex lens, made of a material of rays coming from infinity? µ1
refractive index µ 1 , is placed inside two liquids of (a) Two images are formed
refractive indices µ 2 and µ 3 as shown µ 2 > µ 1 > µ 3 . (b) Continuous image is formed µ2
A wide, parallel beam of light is incident on the lens between focal points of upper and
from the left. The lens will give rise to lower lens
µ2 µ2
(c) One image is formed
µ1 (d) None of the above
83. Two identical thin plano-convex glass lenses
(refractive index 1.5) each having radius of curvature
µ3 µ3
of 20 cm are placed with their convex surfaces in
(a) a single convergent beam contact at the centre. The intervening space is filled
(b) two different convergent beams with oil of refractive index 1.7. The focal length of
(c) two different divergent beams the combination is [CBSE AIPMT 2015]
(d) a convergent and a divergent beam. (a) − 20 cm (b) − 25 cm (c) − 50 cm (d) 50 cm
77. A plano-convex lens is made of material of refractive 84. A convex lens of focal length 1 m and a concave lens
index n. When a small object is placed 30 cm away in of focal length 0.25 m are kept 0.75 m apart. A
front of the curved surface of the lens, an image of parallel beam of light first passes through the convex
double the size of the object is produced. Due to lens, then through the concave lens. Which of the
reflection from the convex surface of the lens, another following option is correct?
faint image is observed at a distance of 10 cm away (a) The final image is formed at 0.5 m away from the
from the lens. which of the following statement(s) is concave lens
(are) true? [JEE Advanced 2016] (b) The final image is formed at focus of the concave lens
(a) The refractive index of the lens is 2.5 (c) The rays after refraction from both the lenses becomes
(b) The radius of curvature of the convex surface is 45 cm parallel to the principal axis
(c) The faint image is erect and real (d) The final image is real and formed at 0.75 m from the
concave lens.
(d) The focal length of the lens is 20 cm
78. A lens is made of flint glass (refractive index = 1.5). 85. Diameter of a plano-convex lens is 6 cm and
When the lens is immersed in a liquid of refractive thickness at the centre is 3 mm. If speed of light in
index 1.25, the focal length material of lens is 2 × 10 8 ms −1 , the focal length of
(a) increases by a factor of 1.25 the lens is [JEE Main 2013]
(b) increases by a factor of 2.5 (a) 15 cm (b) 20 cm (c) 30 cm (d) 10 cm
(c) increases by a factor of 1.2 86. If two + 5 D, lenses are mounted at some distance
(d) decreases by a factor of 1.2 apart, the equivalent power will always be negative, if
79. A concave lens of glass of refractive index 1.5 has the distance is
both surfaces of same radius of curvature R . On (a) greater than 40 cm (b) equal to 10 cm
immersion in a medium of refractive index 1.75, it (c) equal to 40 cm (d) less than 10 cm
will behave as a 87. Two thin lenses, when in contact, produce a
(a) convergent lens of focal length 3.5 R combination of power +10 D. When they are 0.25 m
(b) convergent lens of focal length 3.0 R apart, the power is reduced to + 6 D. The power of the
(c) divergent lens of focal length 3.5 R lenses in dioptres, are
(d) divergent lens of focal length 3.0 R (a) 1 and 9 (b) 2 and 8 (c) 4 and 6 (d) 5 each
88. The power of a biconvex lens is 20 dioptre and the 91. A plano-convex lens fits exactly into a plano-concave
radius of curvature of each surface is 5 cm. Find the lens. Their plane surfaces are parallel to each other. If
refractive index of the material of lens is lenses are made of different materials of refractive
4 3 9 3 indices µ 1 and µ 2 and R is the radius of curvature of
(a) (b) (c) (d)
3 2 8 5 the curved surface of the lenses, then the focal length
of the combination is [CBSE AIPMT 2013]
89. Two thin lenses, when in contact produces a R R
combination of power +10 D. When they are 0.25 m (a) (b)
2( µ 1 + µ 2 ) 2( µ 1 − µ 2 )
apart, the power reduces to + 6 D. The focal length of
the lenses are R 2R
(c) (d)
(a) 0.125 m and 0.5 m (b) 0.125 m and 1 m ( µ1 − µ 2 ) ( µ 2 − µ1 )
(c) 1 m and 0.5 m (d) 0.125 m and 0.4 m 92. A double convex lens whose refractive index is 1.33
90. A convex lens of focal length 40 cm, a concave lens has both radii of curvature of magnitude 10 cm. If an
of focal length 40 cm and a concave lens of focal object is placed at a distance of 5 cm from this lens,
length 15 cm are placed in contact. The power of the the position of the image formed is
combination in dioptre is (a) 7.46 same side of the object
(a) + 1.5 (b) − 1.5 (b) 7.46 opposite side of the object
(c) 14.45 same side of the object
(c) + 6.67 (d) − 6.67
(d) 14.45 opposite side of the object

Topic 4
Prism
93. A ray of light undergoes deviation of 30° when π π
97. A prism of prism angle gives a deviation of . If
incident on an equilateral prism of refractive index 4 4
2. The angle made by the ray inside the prism with the velocity of light in vacuum is 3 × 10 8 ms −1 , then
the base of the prism is find the velocity of light in material of the prism.
(a) 45° (b) 30° (c) 0° (d) 60° (a) 1.25 × 108 ms − 1 (b) 1.33 × 108 ms − 1
94. The refracting angle of a prism is A, and refractive (c) 2.4 × 108 ms − 1 (d) 1.62 × 108 ms − 1
index of the material of the prism is cot (A/2). The
angle of minimum deviation is [CBSE AIPMT 2015] 98. A prism (µ =1.57) gives a deviation of 5.5°. The
(a) 180 ° − 3 A (b) 180° − 2A prism angle is
(c) 90° − A (d) 180 ° + 2 A
(a) 9.64 ° (b) 7.74 ° (c) 5.5° (d) 8.50°
95. A ray of light passes through an equilateral prism such
99. The angle of incidence for a ray of light at a
that, the angle of incidence is equal to the angle of
emergence and the latter is equal to 3/4 the angle of refracting surface of a prism is 45°. The angle of
prism. The angle of deviation is prism is 60°. If the ray suffers minimum deviation
through the prism, the angle of minimum deviation
(a) 25° (b) 30°
and refractive index of the material of the prism
(c) 45° (d) 35°
respectively, are [NEET 2016]
96. A plot of angle of (a) 30°; 2 (b) 45°; 2
Angle of deviation

deviation D versus angle 1 1


50°
of incidence for a (c) 30°; (d) 45°;
2 2
triangular prism is shown
below. 40° 100. In an experiment for determination of refractive index
The angle of incidence for 0 of glass of a prism by i-δ plot, it was found that a ray
30° 45° 60°
which the light ray travels Angle of incidence incident at an angle 35° suffers a deviation of 40° and
parallel to the base is that it emerges at an angle 79°. In that case, which of
(a) 30° (b) 60°
the following is closest to the maximum possible
(c) 45° (d) Data insufficient
value of the refractive index? [JEE Main 2016]
(a) 1.5 (b) 1.6 (c) 1.7 (d) 1.8
101. Which graph truly shows the relation between angle 108. Chromatic aberration is
of incidence and angle of deviation for a prism? (a) formation of a dull image in white light
[JEE Main 2013] (b) formation of a tilted image in white light
δ δ (c) formation of a deformed image in white light

Angle of deviation
Angle of deviation

(d) formation of a coloured image in white light


δm 109. Rainbow is formed due to
(a) δm (b)
(a) refraction (b) reflection
(c) dispersion (d) All of these
i i 110. The primary rainbow is a result of …A… step process.
Angle of incidence Angle of incidence
Here, A refers to
δ δ
Angle of deviation

(a) two (b) one


Angle of deviation

(c) three (d) four


(c) δm (d) 111. When light rays undergoes two internal reflections
δm inside a rain drop. Then, secondary rainbow is formed
due to …A… step process. Here, A refers to
i i (a) one (b) two
Angle of incidence Angle of incidence
(c) three (d) four
102. In vacuum,
(a) red travels faster (b) yellow travels faster 112. Which of the options correctly states the necessary
(c) violet travels faster (d) all colours have same speed condition for formation of rainbow?
(a) Sun should be shining in part of sky while it is raining
103. A monochromatic light is incident on the face of the in opposite part of the sky
prism at an angle of incidence i = 60°. (b) The observer must stand with his back towards the sun
Which of given statement is correct?
(c) The observer must be facing the sun
(a) Dispersion takes place
(d) Both (a) and (b)
(b) Deviation takes place
(c) Both dispersion and deviation takes place 113. The amount of scattering is inversely proportional to
(d) Neither deviation nor dispersion takes place the fourth power of the wavelength. This is known as
104. A white light is incident obliquely on the face of the (a) Rayleigh scattering (b) Maxwell scattering
prism. The light emerging from the other face of the (c) Oersted scattering (d) Reynold scattering
prism consists of 114. At sunset or sunrise, the sun’s rays have to pass
(a) only white light through a larger distance because
(b) a spectrum of light consisting of different components (a) shorter wavelengths are removed by scattering
of colour namely violet, indigo, blue, green, yellow, (b) longer wavelengths are removed by scattering
orange and red
(c) less frequency of scattering wavelength
(c) spectrum consisting of red, blue and green colour
(d) Both (a) and (b)
(d) None of the above
105. White light is incident on one of the refracting surfaces 115. If the ratio of amount of scattering of two light waves
of a prism of angle 5°. If the refractive indices for red is 1 : 4, then the ratio of their wavelengths is
and blue colours are 1.641 and 1.659 respectively, the (a) 2 :1 (b) 2 : 1
angular separation between these two colours when (c) 3 : 1 (d) 2 2 : 1
they emerge out of the prism is
(a) 0.9° (b) 0.09° (c) 1.8° (d) 1.2° 116. Red colour is used for danger signals because
(a) it causes fear
106. When a beam of white light passes through a prism
(b) it undergoes least scattering
then the most deviated colour is
(c) it undergoes maximum scattering
(a) red light (b) violet light (d) None of the above
(c) yellow light (d) Both (a) and (b)
117. The sky would appear red instead of blue, if
107. Thick lenses are not preferred because they (a) atmospheric particles scatter blue light more than red light
(a) are heavy (b) atmospheric particles scatter all colours equally
(b) breaks easily (c) atmospheric particles scatter red light more than the
(c) are costly to manufacture blue light
(d) are prone to chromatic aberration defect (d) the sun was much hotter
Topic 5
Optical Instruments and Defects of Vision
118. The light enters the eye through a curved front 124. Which one is the correct approach to correct the
surface, the …A…. . It passes through the …B… farsighted defect?
which is the central hole in the iris.
Here, A and B refer to (a) P′
(a) cornea, aqueous homour P P′ P
(b) cornea, pupil
(c) pupil, cornea
(d) pupil, aqueous homour (b) P′ P′
P P
119. The retina contains rods and cones which sense Cylindrical
(a) light intensity lens
P P′ P′
(b) color (c) P′
(c) transmit electrical signals
(d) All of the above Image as formed Image as formed
on the retina on the retina
120. When the object is brought closer to the eye, in order
(d) Both (a) and (b)
to maintain the same image lens distance ( ≅ 2.5 cm),
so focal length become shorter by ciliary muscles. 125. Which is the corrective vision for astigmatic eye?
This property of the eye is called …A… .
Here, A refers to (a)
P P
P′
P′
(a) accommodation
(b) vitreous homour
(c) buffer
(d) retina (b) P′ P′
P P
121. Near and far points of human eye are
Cylindrical
(a) 25 cm and infinite lens
(b) 50 cm and 100 cm P P′
(c) P′
(c) 25 cm and 50 cm
(d) 0 cm and 25 cm
Image as formed Image as formed
122. An elderly person tries to read a book at about 25 cm, on the retina on the retina
but the image appears blurred. So, defect of the eye is (d) Both (a) and (b)
called
(a) myopia 126. What focal length should the reading spectacles have
(b) hyperopia for a person for whom the least distance of distinct
(c) presbyopia vision is 50 cm?
(d) Both (a) and (b) (a) + 50 cm (b) − 75 cm
(c) − 50 cm (d) + 75 cm
123. Which is the corrective vision for shortsighted defect?
127. The far point of a myopic person is 80 cm infront of
(a) P′ the eye. What is the power of the lens required to
P P′ P
enable him to see very distant objects clearly?
(a) 2 D (b) 1.50 D
(b) P′ P′ (c) 2.75 D (d) − 1.25 D
P P
Cylindrical 128. The near point of a hypermetropic person is 75 cm
lens
from the eye. What is the power of the lens required
(c) P′ P′
to enable the person to read clearly a book held at
P′ 25 cm from the eye?
Image as formed Image as formed
on the retina on the retina (a) + 2.67 D (b) − 3.42 D
(d) Both (a) and (c) (c) 4.62 D (d) 5.42 D
129. The linear magnification m for the image formed at 138. The distance between the second focal point of the
the near point i. e., 25 cm is objective ( f o ) and first focal point of the eyepiece
 v  d  v  d i. e., f e is called
(a) 1 −  (b) 1 +  (c) 1 +  (d) 1 − 
 f  f  f  f (a) tube length (b) focal length
(c) image distance (d) radius of curvature
130. When an image is at infinity, then the magnification is
139. In an astronomical telescope in normal adjustment a
(a) linear (b) angular
straight black line of length L is drawn on inside part
(c) polarised (d) non-polarised
of objective lens. The eye-piece forms a real image of
131. Identify the wrong descriptions of the below figures this line. The length of this image is I. The
magnification of the telescope is [CBSE AIPMT 2015]
L L L+1 L
F (a) +1 (b) −1 (c) (d)
(i) (ii) I I L−1 I
140. The image formed by an objective of a compound
F microscope is
(iii) (iv) (a) virtual and diminished (b) real and diminished
(c) real and enlarged (d) virtual and enlarged
(a) (i) represents far sightedness
(b) (ii) correction far short sightedness 141. For compound microscope, f o = 1 cm, f e = 2.5 cm.
(c) (iii) represents far sightedness An object is placed at distance 1.2 cm from object
(d) (iv) correction for far sightedness lens. What should be the length of microscope for
132. The nearer point of hypermetropic eye is 20 cm. Find normal adjustment?
the power of lens which can be used for its (a) 8.5 cm (b) 8.3 cm (c) 6.5 cm (d) 6.3 cm
correction? 142. In modern microscopes, the quality of image is
(a) + 2.5 D (b) + 5.0 D (c) − 1.5 D (d) + 1.5 D improved by
(a) minimising optical aberrations in lens
133. A simple magnifier or microscope (b) increasing focal length of objective lens
(a) is a converging lens of small focal length (c) decreasing focal length of objective lens
(b) is a diverging lens of small focal length (d) increasing the radius of curvature of objective lens
(c) gives an erect, magnified and virtual image
143. Astronomical telescope depends on
(d) Both (a) and (c)
(a) its light gathering power
134. A simple microscope has a limited maximum (b) its resolution or resolving power
magnification (c) area of objective lens
(a) greater than 9 (b) lesser than 9 (d) All of the above
(c) equal to 9 (d) Both (b) and (c) 144. An observer looks at a distant tree of height 10 m
135. In order to increase the angular magnification of a with a telescope of magnifying power of 20. To
simple microscope, one should increase observer the tree appears [JEE Main 2016]
(a) the object size (a) 10 times taller
(b) the aperture of the lens (b) 10 times nearer
(c) the focal length of the lens (c) 20 times taller
(d) the power of the lens (d) 20 times nearer

136. A square card of side length 1 mm is being seen through 145. F1 and F2 are focal lengths of objective and eyepiece
a magnifying glass lens of focal length 10 cm. The card respectively, of the telescope. The angular
is placed at a distance of 9 cm from the lens. The magnification of the given telescope is equal to
F1 F2 F1 F2 F1 + F2
apparent area of the card through the lens is (a) (b) (c) (d)
(a) 1 cm2 (b) 0.81 cm2 (c) 0.27 cm2 (d) 0.60 cm2 F2 F1 F1 + F2 F1 F2

137. In a microscope, the focal lengths of two lenses are 146. A astronomical telescope has objective and eyepiece
1.2 cm and 6.25 cm. If an object is placed at 2 cm and of focal lengths 40 cm and 4 cm respectively. To view
the final image is formed at 25 cm from eye lens. The an object 200 cm away from the objective, the lenses
distance between the two lenses is must be separated by a distance [NEET 2016]
(a) 46.0 cm (b) 50.0 cm
(a) 6.00 cm (b) 8.00 cm (c) 7.75 cm (d) 9.25 cm
(c) 54.0 cm (d) 37.3 cm
147. A telescope consists of two thin lenses of focal (c) microscope and telescope both will decrease
lengths 0.3 m and 3 cm, respectively. It is focused on (d) microscope will decrease but that of telescope will
increase
moon which subtends an angle of 0.5° at the
objective. Then, the angle subtended at the eye by the 151. Advantage of reflecting telescopes are
final image will be (a) no chromatic aberration
(a) 5° (b) 0.25° (c) 0.5° (d) 0.35° (b) parabolic reflecting surfaces are used
(c) weighs of mirror are much less than a lens of equivalent
148. An astronomical telescope has an angular optical quality
magnification of magnitude 5 for distant objects. The (d) All of the above
separation between the objective and the eyepiece is
152. Limitation of reflecting telescope is
36 cm and the final image is formed at infinity. The
(a) objective mirror focusses light inside the telescope tube
focal length f o of the objective and the focal length
(b) objective mirror focusses light outside the telescope tube
f e of the eyepiece are (c) objective mirror has large focal length
(a) fo = 45 cm and fe = − 9 cm
(d) tube length is large
(b) fo = − 7.2 cm and fe = 5 cm
(c) fo = 50 cm and fe = 10 cm 153. A reflecting telescope has an objective lens of focal
length 150 cm and an eyepiece of focal length 5.0 cm.
(d) fo = 30 cm and fe = 6 cm What is the separation between the objective and the
149. For a normal eye, the cornea of eye provides a eyepiece?
converging power of 40 D and the least converging (a) 1.25 m (b) 1.55 m (c) 1.3 m (d) 1.8 m
power of the eye lens behind the cornea is 20 D.
154. The largest reflecting telescope in the world are the
Using this information, the distance between the
part of …A… . telescope in Hawaii. Here, A refers to
retina and the cornea, eye lens can be estimated to be
(a) keck (b) cassegrain
[CBSE AIPMT 2013]
(a) 5 cm (b) 2.5 cm (c) 1.67 cm (d) 1.5 cm (c) tetra (d) kistra

150. If the focal length of objective lens is increased, then 155. The largest telescope in India is in
magnifying power of [CBSE AIPMT 2014] (a) Kavalur (b) Andhra Pradesh
(a) microscope will increase but that of telescope decrease (c) Madhya Pradesh (d) Uttar Pradesh
(b) microscope and telescope both will increase

Special Format Questions


I. Assertion and Reason 157. Assertion Refractive index of glass with respect to
■ Directions (Q. Nos. 156-170) In the following air is different for red light and violet light.
questions, a statement of assertion is followed by a Reason Refractive index of a pair of media depends
corresponding statement of reason. Of the following on the wavelength of light used.
statements, choose the correct one. 158. Assertion Propagation of light through an optical
(a) Both Assertion and Reason are correct and Reason is fibre is due to total internal reflection taking place at
the correct explanation of Assertion. the core-clade interface.
(b) Both Assertion and Reason are correct but Reason is
not the correct explanation of Assertion. Reason Refractive index of the material of the core
(c) Assertion is correct but Reason is incorrect. of the optical fibre is greater than that of air.
(d) Assertion is incorrect but Reason is correct. 159. Assertion The refractive index of diamond is 6 and
156. Assertion A ray of light incident along the normal to that of liquid is 3. If the light travels from diamond
the plane mirror retraces its path after reflection from to the liquid, it will initially reflected when the angle
the mirror. of incidence is 30°.
Reason A ray of light along the normal has angle of 1
Reason µ = , where µ is the refractive index of
incidence as π /2 and hence, it retraces its own path sin c
after reflection from mirror. diamond with respect to liquid.
160. Assertion Convergent lens property of converging 170. Assertion The focal length of the objective of the
remains same in mediums. telescope is larger than that of eyepiece.
Reason Property of lens whether the ray is diverging Reason The resolving power of telescope
or converging depends on the surrounding medium. increases when the aperture of objective is small.
161. Assertion By roughening the surface of a glass at
sheet its transparency can be reduced. II. Statement Based Questions Type I
Reason Glass sheet with rough surface absorbs more ■ Directions (Q. Nos. 171-182) In the following
light. questions, a statement I is followed by a
corresponding statement II. Of the following
162. Assertion The focal length of the mirror is f and
statements, choose the correct one.
distance of the object from the focus is u. The (a) Both Statement I and Statement II are correct and
magnification of the mirror is f / u. Statement II is the correct explanation of
Size of image Statement I.
Reason Magnification = (b) Both Statement I and Statement II are correct but
Size of object
Statement II is not the correct explanation of
163. Assertion Thin prisms do not deviate light much. Statement I.
(c) Statement I is correct but Statement II is incorrect.
Reason Thin prism have small angle A and hence,
(d) Statement I is incorrect but Statement II is correct.
Dm (minimum deviation) is also very small as
Dm = [(1 µ 2 − 1) A ], where 1 µ 2 is the refractive index 171. Statement I All distances measured in direction of
light are taken positive.
of prism w.r.t. medium 1.
Statement II Velocity of incident light is positive
164. Assertion A white light on passing through prism according to sign convention used in mechanics.
splits into its component colour such that the red
light emerges nearest to the base of the prism. 172. Statement I If n21 > 1, refracted ray bends towards
Reason Wavelength of red light is more than other the normal and when n21 < 1, refracted ray bends
away from the normal.
component colours and hence, red light deviates least. sin i
Statement II n21 = , angle i is in medium 1
165. Assertion Sunlight reaches us without dispersion in sin r
the form of white light and not as its components. and angle r is in medium 2.
Reason Dispersion takes place due to variation of 173. Statement I Speed of light is different in different
refractive index for different wavelength but in vacuum medium.
the speed of light is independent of wavelength and Statement II Speed of light is more in water than
hence vacuum is a non-dispersive medium. in vacuum.
166. Assertion In case of rainbow, the inner surface of the 174. Statement I Refractive index of turpentine with
water drop get internally reflected. respect to air is more than that of water with respect
Reason The angle between the refracted ray and normal to air.
to the drop surface is greater than the critical angle. Statement II Mass density of turpentine is more
than that of water.
167. Assertion Secondary rainbow is fainter than primary
rainbow. 175. Statement I Critical angle of light while passing
from glass to air is minimum for violet colour.
Reason Secondary rainbow formation is four step
process and hence, the intensity of light is reduced at Statement II The wavelength of violet light is
the second reflection inside the rain drop. greater than the light of other colours.
176. Statement I When white light passes through a
168. Assertion Light from a distant object arriving at the
prism, bending of red component is most but for the
eye lens may get converged at a point in front of the violet it is least.
retina.
Statement II Red colour travels faster than violet
Reason The eye is producing too much divergence in
in glass.
the incident beam.
177. Statement I Rising and setting sun appears
169. Assertion Owls can move freely during night. reddish because of scattering due to dust particles
Reason They have large number of rods on their retina. and air molecules.
Statement II The order of colour in secondary 184. A layer of ice (µ =1.33) lies on a glass plate (µ =1.5).
rainbow is reverse of the order of colours in primary A ray of light makes an angle of incidence 60° on the
rainbow.
surface of ice as shown in the figure below.
178. Statement I If a convex lens of glass is immersed
in water its power decreases. 60°
A Air
Statement II In water, convex lens behaves as Ice
concave lens. r1
r
179. Statement I Power of a lens is defined as p =1/ f , Ice
Glass
f = focal length. r2

Statement II A lens of a high power has large focal


length. With reference to the above figure, match the items in
180. Statement I Astigmatism is occurred when the Column I with terms in Column II and choose the
cornea is not spherical in shape. correct option from the codes given below.
Statement II Astigmatism is corrected by using Column I Column II
cylindrical lens of desired radius of curvature.
A. sin r1 1. (1/ 3 )
181. Statement I Focal length of the eyepiece of
B. sin r2 2. (3 3 )/ 8
telescope is smaller as compared to the objective
which has a larger focal length and aperture. C. Refractive Index of ice with 3. 8/ 9
Statement II For larger focal length of objective respect to glass
than eyepiece, magnification is more in astronomical
A B C A B C
telescopes.
(a) 1 2 3 (b) 2 1 3
182. Statement I According to yerkes observation, the (c) 2 3 1 (d) 1 3 2
objective lens has a diameter of 40 inch (~ 1.02 m).
Statement II To make such large sized lenses, form 185. Match the following Column I and Column II.
images that are free from chromatic aberration and Column I
Column II
distortions. (A simple microscope)
A. The magnifying 1.
III. Matching Type lens is located
such that image is at
h θi
183. With reference to the figure given below, match the the near point.
f θi
items in Column I with terms in Column II and
choose the correct option from the codes given below. Eye focused
at infinity
Normal B. The angle 2.
subtended by the
ay

Eye focused
object, is the same as
Inc

dr

on near point
ide

that at the near point.


cte
nt

fle

u
ra y

D
Re

α
β
Plane mirror C. The object near 3.
the focal point of θ0
Column I Column II the lens; the image is D
far off but closer
A. Angle of reflection 1. ( π − 2α ) than infinity.
B. Value of angle β 2. α
A B C A B C
C. Angle of deviation of the incident ray 3.  π − α (a) 1 2 3 (b) 1 2 3
 
2  (c) 1 3 2 (d) 2 3 1
186. An optical component and an object S placed along
A B C A B C
its optic axis are given in Column I. The distance
(a) 1 2 3 (b) 3 2 1
between the object and the component can be varied.
(c) 2 3 1 (d) 2 1 3
The properties of images are given in Column II.
Match all the properties of images from Column II A B C D A B C D
with the appropriate components given in Column I. (a) 1,3 2,4,5 1,3,5 2,4 (b) 4,1,5 2,1,3 2,5 2
(c) 1,3,5 4,5,1 3,5 1 (d) None of these
Column I Column II
188. A light ray passes through triangular prism with angle
A = 60° and µ =1.5. With reference to the above
A. 1. Real image situation, match the items in Column I with terms in
S
Column II and choose the correct option from the
codes given below.
[Some useful data 2sin −1 (0.75) = 97° ,
B. 2. Virtual image
S sin −1 (2 / 3) = 42° , sin −1 (0.463) = 28° ]
Column I Column II
A. Minimum deviation (Dmin ) 1. 58°
C. S 3. Magnified image
B. Maximum deviation (Dmax ) 2. 37°
C. The angle of refraction r1 when 3. 42°
maximum deviation occurs

D. 4. Image at infinity A B C A B C
S
(a) 1 2 3 (b) 3 2 1
(c) 2 1 3 (d) 3 1 2
A B C D A B C D
(a) 1 3 4 2 (b) 2 1 4 3 189. Match the corresponding entries of Column I with
(c) 3 2 1 4 (d) 3 4 2 1 Column II. (Where, m is the magnification produced
187. Two transparent media of refractive indices µ 1 and µ 3 by the mirror) [NEET 2016]

have a solid lens shaped transparent material of Column I Column II


refractive index µ 2 between them as shown in figures
in Column II. A ray traversing these media is also A. m=−2 1. Convex mirror
shown in the figures. In Column I different m=−
1
B. 2. Concave mirror
relationships between µ 1 , µ 2 and µ 3 are given. Match 2
them to the ray diagram shown in Column II. C. m=+ 2 3. Real image
Column I Column II 1
D. m=+ 4. Virtual image
2
A. µ1 < µ2 1.
µ 3 µ2 µ1 A B C D
(a) 1,3 1,4 1,2 3,4
(b) 1,4 2,3 2,4 2,3
(c) 3,4 2,4 2,3 1,4
B. µ1 > µ2 2.
µ 3 µ2 µ1 (d) 2,3 2,3 2,4 1,4
190. Match Column I with Column II.
Column I (Defect) Column II (Used Lens)
C. µ2 = µ3 3. A. Myopia 1. Cylindrical lens
µ3 µ2 µ1
B. Hypermetropia 2. Concave lens
C. Astigmatism 3. Convex lens
µ1
D. µ2 > µ3 4.
A B C
µ 3 µ2
(a) 2 3 1
(b) 1 2 3
µ1
5. µ3
(c) 3 2 1
(d) 2 1 3
µ2
191. Some laws/processes are given in Column I. Match 194. The angle of refraction in rarer medium for angle of
these with the physical phenomena given in Column II. incidence ic in denser medium is
Column I
(a) 0° (b) 90° (c) 180° (d) 45°
Column II
A. Intensity of light received 1. Radius of aperture (R)
195. If the angle of incidence i in denser medium is greater
by lens than ic ( i. e., i > ic ), then angle of deviation of the
incident ray is
B. Angular magnification 2. Dispersion of lens
(a) ( 90° − ic ) clockwise
C. Length of telescope 3. Focal length f o , f e (b) (180° − 2 i ) clockwise
D. Sharpness of image 4. Spherical aberration (c) (180° + 2 i ) anti-clockwise
(d) Both (b) and (c)
Codes
A B C D A B C D 196. The refractive index of rarer medium is µ r ( = 1.0)
(a) 4,2 1,3,4 2,1 3,1 (b) 1 3 3 1,2,3 and refractive index of denser medium is µ d ( = 1.5).
(c) 2 1 4 4,2,1 (d)1,4,3 4,1 2 1 The critical angle for the pair of media is
 3  2
(a) ic = sin −1   (b) ic = sin −1  
V. Passage Based Questions  2  3
 1
■ Directions (Q. Nos. 192-193) These questions are (c) ic = sin −1   (d) None of these
 3
based on the following situation. Choose the correct
options from those given below. ■ Directions (Q. Nos. 197-198) These questions are
For a rectangular slab, i µ1 based on the following situation. Choose the correct
N
refraction takes place at two A B options from those given below.
r
interfaces (air-glass and Material A has a critical angle θ A and material B has
µ2
glass-air). It is easily seen critical angle θ B (θ B > θ A ). The refractive index of A
from figure that r1 = i , i.e., the r1 and B with respect to air is µ A and µ B , respectively.
emergent ray is parallel to the D C
incident ray, there is no M e µ3 197. Light can be totally internally reflected when it passes
deviation, but it does suffer from
lateral displacement with respect to the incident ray. (a) A to B
(b) B to A
192. If µ 2 > µ 1 > µ 3 , then which of the given relation (c) Either (a) or (b)
is/are correct? (d) total internal reflection is not possible
(a) i = e (b) i ≠ e
198. Critical angle between A and B is
(c) r = r1 (d) Both (b) and (c)  sin θ A   sin θ B 
(a) sin −1   (b) sin −1  
193. If µ 1 = µ 3 < µ 2 , then which of the given relation  sin θ B   sin θ A 
is/are correct? θ  µ 
(c) sin −1  A  (d) sin −1  A 
(a) i = e (b) r ≠ r1  θB  µB 
(c) r = r1 (d) Both (a) and (c)
■ Directions (Q. Nos. 199-201) These questions are
■ Directions (Q. Nos. 194-196) These questions are based on the following situation. Choose the correct
based on the following situation. Choose the correct options from those given below.
options from those given below. A ray of light is incident on the face of the prism
A ray of light travelling in denser medium strikes the (µ = 1.5) at an angle of 60°. The refracting angle of the
interface at an angle i c such that the refracted ray prism is also 60°.
 1
grazes out of the surface. [Some useful data sin −1   = 35°16 ′
 3
(Rarer) µr sin 24° 44 ′ = 0.42]
Refracted ray
199. The angle of emergence is
ic (Denser) µd (a) 30° (b) 45° (c) 39° (d) 60°
y t
ra en
d

200. The angle of deviation is


ci
In

(a) 39° (b) 45° (c) 63° (d) 27°


201. The angle of deviation for angle of incidence i = 60° VI. More than One Option Correct
is D. The angle of incidence other than i for which 208. In case of reflection over a spherical surface, which of
same deviation is produced is these are correct?
(a) 39° (b) 45° (c) 27° (d) 30° (a) Normal is taken as perpendicular of tangent at point of
■ Directions (Q. Nos. 202-203) These questions are incidence
based on the following situation. Choose the correct (b) Perpendicular to incident ray which is perpendicular to
options from those given below. plane of incident ray
(c) Line joining centre of curvature of mirror with point of
Lens Power Aperture incidence
L1 1D 0.1 m (d) Line joining centre of curvature and pole of curved
L2 10 D 0.05 m surface
L3 10 D 0.02 m
209. If i = angle of incidence and r = angle of refraction,
L4 20 D 0.02 m
sin i
then the ratio is
202. Which of the given lens will be preferred as an sin r
eyepiece of an astronomical telescope? (a) a constant for a pair of medium
(a) L 1 (b) L 2 (c) L 3 (d) L 4 (b) is called refractive index of medium 2 with respect to
203. Which of the given lens will be preferred as an medium 1
objective of an astronomical telescope? (c) is called absolute refractive index of medium 2
(a) L1 (b) L 2 (c) L 3 (d) L 4 (d) varies with temperature

■ Directions (Q. Nos. 204-207) These questions are 210. Refractive index of a pair of media depends on
based on the following situation. Choose the correct (a) nature of Ist medium
options from those given below. (b) nature of IInd medium
Total internal reflection is the phenomenon of (c) angle of incidence
reflection of light into denser medium at the interface (d) temperature of medium
of denser medium with a rarer medium. Light must 211. Which one of the following is/are correct relation?
travel from denser to rarer and angle of incidence in 1
denser medium must be greater than critical angle (c) (a) n21 = (b) n32 = n31 × n 12
n12
1
for the pair of media in contact, we can show µ = . n 1a n2 a
sin c (c) n21 = (d) n21 =
n 2a n 1a
204. An astronomical refractive telescope has an objective
of focal length 20 m and an eyepiece of focal length 212. Which of the following statements is/are correct about
2 cm. Which of the following statement is not correct? a convex lens?
(a) The length of the telescope tube is 20.02 m (a) Convex lens is converging for light for all wavelengths
(b) The magnification is 1000 (b) For virtual object, the image is also virtual
(c) The image formed is inverted (c) For real object, the image is always real
(d) An objective of a larger aperture will increase the (d) None of the above
brightness and reduce chromatic aberration of the image
205. Critical angle for water air interface is 48.6°. What is 213. A point object is at 30 cm from a convex glass lens
the refractive index of water?  3
 µ s =  of focal length 20 cm. The final image of
(a) 1 (b) 3/2 (c) 4/3 (d) 3/4  2
206. Light is travelling from air to water at ∠i = 50°, which object will be formed at infinity, if
is greater than critical angle for air water interface. (a) another concave lens of focal length 60 cm is placed in
What fraction of light will be totally reflected? contact with the previous lens
(a) 100% (b) 50% (b) another convex lens of focal length 60 cm is placed at a
distance of 30 cm from the first lens
(c) 25% (d) None of these
(c) the whole system is immersed in a liquid of refractive
207. Critical angle for glass air interface where refractive index 4/3
index µ of glass is 3/2 is (d) the whole system is immersed in a liquid of refractive
(a) 41.8° (b) 60° (c) 30° (d) 44.3° index 9/8
214. Sunlight reaches to us in composite form and not in 215. Which of the following statement(s) is/are correct?
it’s constituent colours because (a) In compound microscope, objective lens forms a real,
(a) vacuum is non-dispersive. inverted, magnified image of the object
(b) speed of all colours is same in vacuum. (b) Eyepiece lens, produces the final image which is
(c) light behaves like a particle in vacuum. enlarged and virtual
(d) light travels in a straight line in vacuum. (c) The final image is inverted with respect to the original
object
(d) None of the above

NCERT & NCERT Exemplar Questions


NCERT 221. Double-convex lenses are to be manufactured from a
216. A small candle 2.5 cm in size is placed 27 cm infront glass of refractive index 1.55, with both faces of the
of a concave mirror of radius of curvature 36 cm. At same radius of curvature. What is the radius of
what distance from the mirror should a screen be curvature required if the focal length is to be 20 cm?
placed in order to receive a sharp image? What is the (a) 20 cm (b) 21 cm
size of the image? (c) 22 cm (d) 23 cm
(a) 54 cm, 5 cm (b) − 54 cm, − 5 cm 222. A beam of light converges at a point P. Now a lens is
(c) − 27 cm, 2.5 cm (d) − 54 cm, − 2.5 cm placed in the path of the convergent beam 12 cm from
217. A 4.5 cm needle is placed 12 cm away from a convex P. At what point does the beam converge if the lens is
mirror of focal length 15 cm. Find the magnification a convex lens of focal length 20 cm?
of this image. (a) 7.5 cm (b) 8.5 cm
5 9 (c) 9.5 cm (d) 6.5 cm
(a) (b)
9 5 ■ Directions (Q. Nos. 223-224) Answer the following
(c) 5.5 (d) None of these questions which are based on given paragraph. Choose
218. A tank is filled with water to a height of 12.5 cm. The the correct option from those given below.
apparent depth of a needle lying at the bottom of the An object of size 3.0 cm is placed 14 cm infront of a
tank is measured by a microscope to be 9.4 cm. What concave lens of focal length 21 cm. The image distance
is the refractive index of water? If water is replaced is v, magnification m and image height I.
by a liquid of refractive index 1.63 upto the same
height, by what distance would the microscope have 223. With reference to the above situation, match the items
to be moved to focus on the needle again? in Column I with terms in Column II and choose the
(a) 1.32, 7.7 cm (b) 1.63, 7.7 cm option given below.
(c) 1.32, 1.7 cm (d) 1.63, 1.7 cm Column I Column II
219. A small bulb is placed at the bottom of a tank A v (Image distance in cm) 1. 0.6
containing water to a depth of 80 cm. What is the area
B I (Image height in cm) 2. 8.4
of the surface of water through which light from the
bulb can emerge out? Refractive index of water is C m (magnification) 3. 1.8
1.33. (Consider the bulb to be a point source.)
A B C A B C
(a) 3.58 m 2 (b) 2.58 m 2 (c) 6 m 2 (d) 4 m 2
(a) 1 2 3 (b) 3 2 1
220. A prism is made of glass of unknown refractive (c) 2 1 3 (d) 2 3 1
index. A parallel beam of light is incident on a face of
the prism. The angle of minimum deviation is 224. Which of the following statement correctly describes
measured to be 40°. What is the refractive index of the object?
the material of the prism? The refracting angle of the I. The image is virtual and erect.
prism is 60°. If the prism is placed in water (refractive II. Image is diminished.
index 1.33), predict the new angle of minimum III. If the object is moved further away from the lens
deviation of a parallel beam of light. betweenO and F, the image moves towards the lens
(a) 1.53, 10°20' (b) 1.2, 35°10' (never beyond focus) and the size of image decreases
(c) 1.53, 35°10' (d) 1.2, 10°20' gradually.
Choose the correct answer from the options given 232. A screen is placed 90 cm from an object, the image of
below. the object on the screen is formed by a convex lens at
(a) I and III (b) I and II (c) II and III (d) I, II and III two different locations separated by 20 cm. Determine
225. The plano-convex lens of focal length 20 cm and the focal length of the lens.
30 cm are placed together to from a double convex (a) 20.5 cm (b) 19.5 cm
lens. The final focal length will be (c) 23.2 cm (d) 21.4 cm

(a) 12 cm (b) 60 cm (c) 20 cm (d) 30 cm 233. At what angle should a ray of light be incident on the
face of a prism of refracting angle 60° so that it just
226. A person with a normal near point 25 cm using a suffers total internal reflection at the other face? The
compound microscope with an objective of focal refractive index of the material of the prism is 1.524.
length 8.0 mm and eyepiece of focal length 2.5 cm (a) 30°75′ (b) 29°75′
can bring an object placed at 9.0 mm from the (c) 28°75′ (d) 27°75′
objective in sharp focus. What is the separation
between the two lenses? How much is the magnifying 234. For a normal eye, the far point is at infinity and the
power of the microscope? near point of distinct vision is about 25 cm infront of
(a) 7.2, 89 (b) 9.4, 88 (c) 8.6, 90 (d) 9.4, 75 the eye. The cornea of the eye provides a converging
227. A small telescope has an objective lens of focal power of about 40 D and the least converging power
length 144 cm and an eyepiece of focal length 6.0 cm. of the eyelens behind the cornea is about 20 D. From
What is the magnifying power of the telescope? What this rough data estimate the range of accommodation
is the separation between the objective and the (i.e., range of converging power of its eyelens) of a
eyepiece? normal eye.
(a) 20-24 D (b) 18-22 D
(a) − 24, 150 cm (b) − 25, 150 cm
(c) 64-68 D (d) 35-40 D
(c) − 24, 160 cm (d) − 25, 160 cm
228. A giant refracting telescope at an observatory has an 235. A myopic person has been using spectacles of power
objective lens of focal length 15 m. If an eyepiece of −1.0 D for distant vision. During old age he also
focal length 1.0 cm is used, what is angular needs to use separate reading glass of power + 2.0 D.
magnification of the telescope? Find the near point of this person.
(a) + 50 cm (b) − 50 cm
(a) 1200 (b) 1000 (c) 1300 (d) 1500
(c) − 100 cm (d) + 100 cm
229. If in the question 228, this telescope is used to view
the moon, what is the diameter of the image of the 236. A person looking at a person wearing a shirt with a
moon formed by the objective lens? The diameter of pattern comprising vertical and horizontal lines is
able to see the vertical lines more distinctly than the
the moon is 3.48 ×10 6 m and the radius of lunar orbit
horizontal ones. What is this defect called? How is
is 3.48 ×10 8 m. such a defect of vision corrected?
(a) 13.5 cm (b) 13.2 cm (c) 13.7 cm (d) 13.9 cm (a) Astigmatism, convex lens
230. A small pin fixed on a table top is viewed from above (b) Myopia, concave lens
from a distance of 50 cm. By what distance would the (c) Astigmatism, cylindrical lens
pin appear to be raised if it is viewed from the same (d) Presbyopia, cylindrical lens
point through a 15 cm thick glass slab held parallel to 237. A man with normal near point (25 cm) reads a book
the table? Refractive index of glass =1.5. Does the with small print using a magnifying glass : a thin
answer depend on the location of the slab? convex lens of focal length 5 cm. What is the closest
(a) 5 cm, yes and the farthest distance at which he can read the
(b) 5 cm, no book when viewing through the magnifying glass?
(c) 10 cm, data insufficient (a) − 4.2 cm, − 5 cm
(d) None of the above (b) 4.2 cm, 5 cm
231. The image of a small electric bulb fixed on the wall to (c) − 4.2 cm, 5 cm
a room is to be obtained on the opposite wall 3 m (b) 4.2 cm, − 5 cm
away by means of a large convex lens. What is the 238. What is the maximum and the minimum angular
maximum possible focal length of the lens required magnification (magnifying power) possible using the
for the purpose? above simple microscope?
(a) 3 m (b) 0.75 m (c) 0.45 m (d) 0.25 m (a) 6, 2 (b) 7, 5 (c) 6, 5 (d) 6, 3
239. A card sheet divided into squares each of size 1 mm 2 244. Figure shows an equiconvex lens Q P P′ Q′
is being viewed at a distance of 9 cm held close to the (of refractive index 1.50) in contact
eye. with a liquid layer on top of a plane
(i) What is the magnification produced by the lens? mirror. A small needle with its tip on
How much is the area of each square in the virtual the principal axis is moved along the
image? axis until its inverted image is found
(ii) What is the angular magnification (magnifying at the position of the needle. The
power) of the lens? distance of the needle from the lens
(a) 2.8, 10 cm2 , 2.8 (b) 10, 1 cm2 , 2.8
is measured to be 45 cm. The liquid is removed and the
experiment is repeated. The new distance is measured to
(c) 10, 1 cm2 , 10 (b) 10, 100 cm2 , 2.8 be 30 cm. What is the refractive index of the liquid?
240. An angular magnification (magnifying power) of (a) 1.42 (b) 1.50 (c) 1.26 (d) 1.33
30 X is desired using an objective of focal length
1.25 cm and an eyepiece of focal length 5 cm. How NCERT Exemplar
will you set up the compound microscope? 245. A ray of light incident at an angle θ on a refracting face
(a) u0 = − 1.5 cm, v0 = 7.5 cm, ue = − 4.17 cm of a prism emerges from the other face normally. If the
(b) u0 = 1.5 cm, v0 = −7.5 cm, ue = 4.17 cm angle of the prism is 5° and the prism is made of a
(c) u0 = − 1.5 cm, v0 = − 7.5 cm, ue = − 4.17 cm material of refractive index 1.5, the angle of incidence
(d) u0 = − 1.5 cm, v0 = 7.5 cm, ue = 4.17 cm is
241. A small telescope has an objective lens of focal (a) 7.5° (b) 5° (c) 15° (d) 2.5°
length 140 cm and an eyepiece of focal length
5.0 cm. What is the magnifying power of the 246. A short pulse of white light is incident from air to a
telescope for viewing distant objects when glass slab at normal incidence. After travelling through
(i) the telescope is in normal adjustment (i.e., when the the slab, the first colour to emerge is
final image is at infinity). (a) blue (b) green (c) violet (d) red
(ii) the final image is formed at the least distance of 247. An object approaches a convergent lens from the left of
distinct vision (25 cm). the lens with a uniform speed 5 ms −1 and stops at the
(a) 14, 28 (b) 5, 1.2 focus. The image
(c) 28, 28.7 (d) 28, 33.6 (a) moves away from the lens with an uniform speed 5 ms − 1
242. (i) If the telescope used in Q. 241 is used to view a (b) moves away from the lens with an uniform acceleration
100 m tall tower 3 km away, what is the height of the (c) moves away from the lens with a non- uniform
image of the tower formed by the objective lens? acceleration
(ii) What is the height of the final image of the tower if it (d) moves towards the lens with a non-uniform acceleration
is formed at 25 cm? 248. A passenger in an aeroplane shall
(a) 28 cm, 4.67 cm (b) 5.32 cm, 14 cm (a) never see a rainbow
(c) 4.67 cm, 28 cm (d) 6 cm, 30 cm (b) may see a primary and a secondary rainbow as concentric
243. A Cassegrain telescope uses two mirrors as shown circles
in figure. Such a telescope is built with the mirrors (c) may see a primary and a secondary rainbow as concentric
arcs
20 mm apart. If the radius of curvature of the large
(d) shall never see a secondary rainbow
mirror is 220 mm and the small mirror is 140 mm,
where will the final image of an object at infinity be? 249. You are given four sources of light each one providing
a light of a single colour-red, blue, green and yellow.
Suppose the angle of refraction for a beam of yellow
Objective
mirror light corresponding to a particular angle of incidence at
the interface of two media is 90°. Which of the
Secondary following statements is correct, if the source of yellow
mirror light is replaced with that of other lights without
Eyepiece changing the angle of incidence?
(a) The beam of red light would undergo total internal
reflection.
(a) 315 mm (b) 415 mm (b) The beam of red light would bend towards normal while it
(c) 215 mm (d) 115 cm gets refracted through the second medium.
(c) The beam of blue light would undergo total internal 254. There are certain materials developed in laboratories
reflection which have a negative refractive index (figure shown
(d) The beam of green light would bend away from the below). A ray incident from air (medium 1) into such
normal as it gets refracted through the second medium a medium (medium 2) shall follow a path given by
250. The radius of curvature of the curved surface of a
plano-convex lens is 20 cm. If the refractive index of i 1 i 1
the material of the lens be 1.5, it will (a) (b)
(a) act as a convex lens only for the objects that lie on its r 2 r 2
curved side
(b) act as a concave lens for the objects that lie on its
curved side
(c) act as a convex lens irrespective of the side on which
the object lies i r 1 1
(d) act as a concave lens irrespective of side on which the (c) (d)
object lies 2 2

251. The phenomena involved in the reflection of


radio waves by ionosphere is similar to
255. Consider an extended object immersed in water
(a) reflection of light by a plane mirror
contained in a plane trough. When seen from close to
(b) total internal reflection of light in air during a mirage
the edge of the trough the object looks distorted
(c) dispersion of light by water molecules during the
formation of a rainbow because
(d) scattering of light by the particles of air (a) the apparent depth of the points close to the edge are
nearer to the surface of the water compared to the points
252. The direction of ray of light incident on a concave away from the edge
mirror is shown by PQ while directions in which the (b) the angle subtended by the image of the object at the
ray would travel after reflection is shown by four rays eye is smaller, than the actual angle subtended by the
marked 1,2,3 and 4 as shown in figure. Which of the object in air
four rays correctly shows the direction of reflected ray? (c) some of the points of the object far away from the edge
may not be visible because of total internal reflection
1
Q (d) All of the above
2 4
256. A rectangular block of glass ABCD A B
has a refractive index 1.6. A pin is
C F placed midway on the face AB figure.
3 When observed from the face AD.
P The pin shall D C
(a) appear to be near B
(a) 1 (b) 2 (b) appear to be near D
(c) 3 (d) 4 (c) appear to be at the centre of AD
253. The optical density of turpentine is higher than that of (d) not be seen at all
water while its mass density is lower. Figure shows a 257. A magnifying glass is used, as the object to be viewed
layer of turpentine floating over water in a container. can be brought closer to the eye than the normal near
For which one of the four rays incident on turpentine point. This results in
in figure, the path shown is correct? (a) a larger angle to be subtended by the object at the eye
3 4 and hence, viewed in greater detail
2
1 (b) the formation of a virtual and erect image
Air (c) increase in the field of view
(d) infinite magnification at the near point
Turpentine 258. The near vision of an average is 25 cm. To view an
object with an angular magnification of 10, what
Water should be the power of the microscope?
(a) 1 (b) 2 (c) 3 (d) 4 (a) 10 D (b) 20 D (c) 40 D (d) 30 D
259. Three immiscible liquids of densities d1 > d 2 > d 3 and 262. A jar of height h is filled with a transparent liquid of
refractive indices µ 1 > µ 2 > µ 3 are put in a beaker. refractive index µ shown in figure. At the centre of
h the jar on the bottom surface is a dot. Find the
The height of each liquid column is . A dot is made minimum diameter of a disc, such that when placed
3
at the bottom of the beaker. For near normal vision, on the top surface symmetrically about the centre, the
find the apparent depth of the dot? dot is invisible.
d
h 1 1 1  1 1 1
(a)  + +  (b) 3h  + − 
3  µ1 µ 2 µ 3   µ1 µ 2 µ 3 
h 1 1 1  1 1 1
(c)  − +  (d) 3h  − −  h
3  µ1 µ 2 µ 3   µ1 µ 2 µ 3  i i

260. For a glass prism (µ = 3 ), the angle of minimum


deviation is equal to the angle of the prism. Find the
angle of the prism. 2h h h 2 µ2 −1
(a) (b) (c) (d)
(a) 40° (b) 45° µ2 −1 µ2 −1 2 µ2 −1 h
(c) 60° (d) 75°
263. A myopic adult has a far point at 0.1 m. His power of
261. A thin convex lens of focal length 25 cm is cut into accomodation is 4 D.
two pieces 0.5 cm above the principal axis. The top (i) What power lenses are required to see distant objects?
part is placed at (0, 0) and an object placed at
(ii) What is his near point without glasses?
(−50 cm,0). The coordinates of the image are
(iii) What is his near point with glasses? (Take the image
(a) (50 cm, −1cm)
distance from the lens of the eye to the retina to be
(b) (50 cm, +1cm) 2 cm.)
(c) (−50 cm, +1cm) (a) − 10 D, 0.07 m, 0.25 m (b) 50 D, 0.25 m, 0.07 m
(d) (−50 cm, −1cm) (c) 10 D, 0.07 m, 0.25 m (d) − 50 D, 0.07 m, 0.25 m

Answers
1. (a) 2. (a) 3. (b) 4. (d) 5. (d) 6. (d) 7. (d) 8. (a) 9. (c) 10. (a) 11. (a) 12. (c) 13. (c) 14. (b) 15. (d)
16. (a) 17. (b) 18. (d) 19. (d) 20. (a) 21. (a) 22. (d) 23. (c) 24. (a) 25. (d) 26. (c) 27. (c) 28. (d) 29. (d) 30. (c)
31. (b) 32. (d) 33. (c) 34. (c) 35. (b) 36. (a) 37. (c) 38. (b) 39. (a) 40. (a) 41. (c) 42. (c) 43. (c) 44. (c) 45. (d)
46. (d) 47. (c) 48. (b) 49. (a) 50. (b) 51. (c) 52. (a) 53. (a) 54. (c) 55. (b) 56. (c) 57. (d) 58. (a) 59. (c) 60. (b)
61. (c) 62. (d) 63. (d) 64. (a) 65. (c) 66. (a) 67. (a) 68. (b) 69. (d) 70. (c) 71. (a) 72. (c) 73. (c) 74. (b) 75. (c)
76. (d) 77. (a, 78. (b) 79. (a) 80. (d) 81. (b) 82. (a) 83. (c) 84. (c) 85. (c) 86. (a) 87. (b) 88. (b) 89. (a) 90. (d)
d)
91. (c) 92. (a) 93. (c) 94. (b) 95. (b) 96. (c) 97. (d) 98. (a) 99. (a)
103. (b)
100. (a) 101. (d) 102. (d) 104. (b) 105. (b)
106. (b) 107. (d) 108. (d) 109. (d) 110. (c) 111. (d) 112. (d) 113. (a) 114. (a)
118. (b)
115. (a) 116. (b) 117. (c) 119. (d) 120. (a)
121. (a) 122. (c) 123. (a) 124. (b) 125. (c) 126. (a) 127. (d) 128. (a) 129. (b)
133. (d)
130. (b) 131. (a) 132. (b) 134. (d) 135. (d)
136. (a) 137. (b) 138. (a) 139. (d) 140. (c) 141. (b) 142. (a) 143. (d) 144. (d)
148. (d)
145. (a) 146. (c) 147. (a) 149. (c) 150. (d)
151. (d) 152. (a) 153. (b) 154. (a) 155. (a) 156. (c) 157. (a) 158. (b) 159. (a)
163. (a)
160. (d) 161. (c) 162. (d) 164. (d) 165. (a)
166. (a) 167. (a) 168. (c) 169. (c) 170. (c) 171. (c) 172. (a) 173. (c) 174. (c)
178. (c)
175. (c) 176. (d) 177. (b) 179. (c) 180. (a)
181. (a) 182. (a) 183. (c) 184. (b) 185. (d) 186. (c) 187. (a) 188. (c) 189. (d)
193. (d)
190. (a) 191. (b) 192. (d) 194. (b) 195. (d)
196. (b) 197. (a) 198. (a) 199. (c) 200. (a) 201. (a) 202. (d) 203. (a) 204. (d)
208. (a,c
205. (c) 206. (d) 207. (a) 209.(a,b, 210.(a,b,
) d) d)
211. (a,b, 212. (a, 213. (a,d 214. (a, 215. (a,b, 216. (b) 217. (a) 218. (c) 219. (b) 220. (a) 221. (c) 222. (a) 223. (d) 224. (d) 225. (a)
d) b) ) b) c)
226. (b) 227. (a) 228. (d) 229. (c) 230. (b) 231. (b) 232. (d) 233. (b) 234. (a) 235. (b) 236. (c) 237. (a) 238. (c) 239. (b) 240. (a)
Hints and Explanations
1. (a) Nature has endowed the human eye (retina) with the 10. (a) Given,
sensitivity to detect electromagnetic waves within a small Incident ray is represented by vector A
range of the electromagnetic spectrum.
A = 2i$ − 3$j + 4 k$
2. (a) Light travels with a speed of c = 2.99792458 × 108 ms −1 .
For many purposes, it suffices to take c = 3 × 108 ms −1 . The Similarly, normal is represented by
speed of light in vacuum is the highest speed attainable in B = 3i$ − 6$j + 2 k$
nature. Let reflected ray be represented by vector R .
3. (b) A light wave can be considered to travel from one point R=?
to another along a straight line joining them. The path is Here, normal can be represented by any vector pointing up
called a ray of light and bundle of such rays constitutes a from the mirror or downwards.
beam of light. We can verify, A ⋅ B = 6 + 18 + 8 = 32 (+ve)
4. (d) The wavelength of light is very small compared to the Since, A ⋅ B > 0, angle between incident
size of ordinary objects that we encounter commonly
$ = 2i − 3 j + 4 k
$ $ $
(generally of the order of a few centimetres or larger). Unit vector along A , A …(i)
29
5. (d) According to law of reflection, the angle of reflection
(i.e., the angle between reflected ray and the normal to the Unit vector along B,

$ = 3i − 6 j + 2k = 3i − 6 j + 2k
reflecting surface or the mirror) equals the angle of $ $ $ $ $ $
incidence (angle between incident ray and the normal). B …(ii)
49 7
Hence, θ1 = θ 2
Let ON and OM represent the unit vectors , hence
Inc
ide Normal
ay ON = OM − MN (from triangle law) …(iii)
nt r
ray c ted |MN | = 2 (projection length of vector ON along PN )
fl e
Re
θ1 θ2 = 2 (A $ = 2 32
$ ⋅ B)
7 29
From Eq. (iii), R = A − 2 ( A ⋅ B
$ $ $ $ )B $
Mirror
Substituting the values of Eqs. (i) and (ii), we get
6. (d) Normal, incident ray and reflected ray are coplanar lines.  2i$ − 3$j + 4 k$   32   3i$ − 6$j + 2k$ 
R=  − 2   
 29   7 29   7 
7. (d) Laws of reflection are valid in case of regular reflection
and over a curved surface.  49 ( 2i$ − 3$j + 4 k$ ) − (192i$ − 384 $j + 128 k$ )
8. (a) Velocity of object, vob = 3i$ + 4 $j
= 
 49 29 
Velocity of image, vimage = 3$i − 4 $j
 98$i − 147$j + 196 k$ − 192i$ + 384 $j − 128 k$ 
Relative velocity of image with respect to its object = 
 49 29 
v rel = vimage − vob = ( 3i$ − 4 $j ) − ( 3i$ + 4 $j ) = − 8 $j
 − 94 $i + 237$j + 68 k$ 
9. (c) From the law of reflection,
= 
 49 29 
Normal
11. (a) Normal
y
In

ra
c

i
id

ed

θ
en

θ 1
ct
t


fle
ra

2
y

Re

i
O
√3 incident ray

2
Let, angle of incidence = angle of reflection = θ
 1
Given, incident ray is perpendicular to reflected ray (from  
 2 1
figure) tan i = = ⇒ i = 30°
 3 3
⇒ θ = 90° ⇒ θ = 45°  
So, angle of incidence = 45°  2
12. (c) Let θ be the angle between mirrors M 1 and M 2 . a−x
tan θ = ...(ii)
From figure, ABCD is a parallelogram. y
∴ ∠BCA = 90° − ( 90° − θ ) = θ tan θ =
a
...(iii)
Reflected ray a− y
(from M2)
C D From Eqs. (i) and (ii), we get
r2
2x a − x
i2 = ⇒ 2xy = a 2 − xa ...(iv)

(froment ray
a y

M1 )
M2 From Eqs. (ii) and (iii), we get

Incid
a−x a
r1
i1 =
y a− y
θ
B
M1 A a 2 − ya − xa + xy = ya ⇒
Also, ∠BCA = ∠DAC ( AD || BC ) a 2 − xa − ya + xy = ya
⇒ i1 + r1 = θ ⇒ i1 = r1 = θ/ 2 3xy = 2ay [Using Eq. (iv)]
∠CAB = ( 90° − θ / 2 ) 2a
So, x=
From ∆ABC, θ + θ + ( 90° − θ / 2 ) = 180° 3
3θ 90° × 2 Substituting this value of x in Eq. (i), we get
⇒ = 90° ⇒ θ = = 60° ( 2a / 3 ) 4
2 3 tan θ = =
B (a / 2) 3
13. (c)
Mirror (M1) Mirror (M2)
1 3
Normal
θ
Normal ∴ cot θ = = or θ = cot −1 ( 0.75 )
tan θ 4
N
M i i1 Q 15. (d) In a concave mirror, the reflected rays converge at a
2i
i i1 point F on the principal axis while in a convex mirror,the
reflected rays appear to diverge from a point F on its
A C
principal axis. This point F is called focal point or focus. So,
L S
in all cases F is the focus.
Let θ be angle between the mirrors M 1 and M 2 .
16. (a) Parallel beam passes through focus after reflection. This
Since, rays LM and NS are parallel to each other.
can be shown in the figure given below.
∠LMN = ∠SNQ = 2i
⇒ 2i1 + 2i = 180° .⇒ i1 + i = 90° …(i)
Also, in ∆MBN , θ + ( 90 − i ) + ( 90 − i1 ) = 180°
⇒ θ = i1 + i …(ii) A (F )
On comparing Eqs. (i) and (ii), we get
θ = 90°
14. (b) The ray starting from point M at an angle θ reaches the
corner D at the right along a parallel path. Let a be the 17. (b) As we know, angle i = 0° and angle r = 0° , when light
length of the side. ray is passes through centre of curvature of a spherical
mirror is reflected such that it trace back its path.
y a–y
B C 18. (d) If lower half of a concave mirror is blackened, then
θ θ
a–x image will now only half of the object, but taking the laws
θ of reflection to be true for all points of the remaining part of
θ the mirror, the image will be that of the whole object.
However, as the area of the reflecting surface has been
x reduced, the intensity of the image will be low i.e., half.
−15
θ θ 19. (d) The focal length, f = = −7.5 cm
A M D 2
a/2 a/2 The object distance i.e., u = −10 cm, v = ?
1 1 1
From figure, According to mirror formula, we get + =
x v u f
tan θ = ...(i) 1 1 1 10 × 7.5
(a / 2) ⇒ + = ⇒ v= = −30 cm
v ( −10 ) ( −7.5 ) −2.5
The image is 30 cm from the mirror on the same side as the 23. (c) According to question,the ray diagram will be
−v ( −30 )
object. Also, magnification, m = =− = −3.
u ( −10 )
Thus, the image is magnified, real and inverted.
fv
20. (a) From the mirror equation, we get v = f2 2f1
u− f
So, distance d between convex lens and concave mirror is given by
For convex mirror, since R = 2 m, f = 1 m.
d = 2 f1 + f2
( −39 ) × 1 39
Then, u = − 39 m, v = = m 24. (a) Here, h1 = 2 cm, u = − 16 cm h2 = − 3 cm
−39 − 1 40
(since image is real and inverted)
Since, the jogger moves at a constant speed of 5 ms −1 , after 1s
− h2 v
the position of the image v (for u = − 39 + 5 = − 34 ) is (34/35) ∴ m= =
m. h1 u
The shift in the position of image in 1s is − h2 3
⇒ v= u = × ( − 16 ) = − 24 cm
39 34 1365 − 1360 5 1 h1 2
− = = = m
40 35 1400 1400 280 1 1 1 1 1
Now, = + =− −
Therefore, the average speed of the image when the jogger is f v u 24 16
between 39 m and 34 m from the mirror, is (1/280) ms −1 . − 2− 3 − 5 − 48
⇒ = ⇒ f = = − 9.6 cm
21. (a) Let AB represents the boy with height AB = 1.5 m and 48 48 5
C represents the eye level. 25. (d) By mirror formula image distance of A
B 1 1 1
+ = ⇒ f = − 10 cm
0.12 m

i v u f
F i N
f=–10 cm
C (Mirror)
1.38 m

i1
E i1 M A C

A
Let MN represents the minimum height of the mirror. 1 1 1 1 1
+ = − = ⇒ v A = − 15 cm
⇒ MN = FC + CE vA u f ( −30 ) −10
 BC   AC  Also, image distance of C ⇒ vC = − 20 cm
=  +  = 0.06 + 0.69
 2   2  The length of image = | v A − VC | = | − 15 − ( −20 )| = 15 cm
∴ MN = 0.75 m 26. (a) For spherical mirror, f =
R
22. (d) We know, from the mirror formula, 2
here, R = 20 cm, m = 2.5
 uf 
v=  …(i) R 20
u − f ∴ f = = = 10 cm
2 2
On differentiating Eq. (i) both sides w.r.t. u, we get 1 1 1
Now, = +
dv ( u − f ) f − ( uf ) dv u f − f 2 − uf f v u
= ⇒ =
du ( u − f )2 du ( u − f )2 −v
and magnification m = or v = − mu
2 u
dv f2  f   1  1
⇒ =− ⇒ dv = −   du ∴
1 1
= −
1
⇒ u = f 1 −  = 10 1 −
( u − f )2 u − f  = 6 cm
du f u mu  m  2.5
2
 f  27. (c) Apparent depth of the object =
33.2
|dv | =   |du | …(ii) cm
u− f  1.33
33.5
Here, image size = |dv | Distance of object from the mirror = 15 + = 40 cm
1.33
object size = | du | = b 25
On substituting values in Eq. (ii), we get Similarly, distance of image from the mirror = 15 +
1.33
Image size = b ( f / u − f )2 . = 33.8cm
For concave mirror, u = − 40 cm or v = − 33.8 cm 32. (d) Given, i = angle of incidence = 60° and ∠BMC = 90°
Applying mirror formula,
A B
uv ( −40) ( −33.8) Inc
f = = = − 18.3 cm ide r ay
u + v ( − 40 − 33.8) nt ted
ray 60° 60° flec
Air Re
28. (d) The incident ray, the refracted ray and the normal to the
interface at the point of incidence, all lie in the same plane. Slab M

Re
r

fr a
Inc
ay

cte
ide
nt edr C

dr
ray c t
fle R

ay
Re
θ θ
⇒ ∠CMR = r = angle of refraction = 180° − ( 60° + 90° )
⇒ r = 30°
Reflecting surface
sin i µ slab
From Snell’s law, =
sin r µ air
Refracted ray
sin 60° µ slab
⇒ =
29. (d) From Snell’s law of refraction, sin 30° 1.0
sin i
a
µg == constant …(i)  3
sin r ∴ µ slab =   × 2 = 3
Since, angle of incidence increase, the angle of refraction has  2
 sin i  33. (c) Given, i = 45°
to increase. So, that the ratio   is a constant according to
 sin r
Incident ray Reflected ray
Eq. (i).
Air
30. (c) For same angle of incidence, the angle of refraction is i r (µa = 1)
minimum in medium R
θ (µg = √2)
µ1 µ2 µ3 µ4
r′ Refracted ray
D
r2 e
r2 C
r1 sin i µ g
r1 From Snell’s law, =
i B sin r′ µ a
A
sin 45° 2
sin i ⇒ =
As, µ= …(i) sin r′ 1
sin r
1  1
µ∝
1 sin r′ = ⇒ r′ = sin −1   = 30°
or 2  2
sin r
⇒ µ is maximum for R, since r is minimum and From diagram, r + θ + r′ = 180°
hence, sin r is minimum. i + θ + 30° = 180° (Q i = r)
c 45 + θ + 30° = 180°
Also, µ= …(ii)
v θ = 180° − 75° = 105°
Therefore, if µ is maximum, v is minimum i.e., velocity of
light is minimum in medium R and order of velocity will be Hence, the angle between reflected and refracted ray is 105°.
vP > vQ > vR sin 50°
34. (c) a
µg = …(i)
31. (b) When a ray of light passes through the glass slab, the sin 40°
emergent rays are parallel to the incident rays. sin 50°
a
µw = …(ii)
α sin 45°
i i
sin 45°
w
µg = …(iii)
sin θ
α n t a
µ w × wµ g = a µ g
sin 50° sin 45° sin 50°
× = (Using Eq. (i), (ii) and (iii)
e=
i

sin 45° sin θ sin 40°


e=

Here, from figure, i = e. sin θ = sin 40°


So, divergent angle of the emergent beam is also same i.e., α. θ = 40°
35. (b) For a rectangular slab, refraction takes place at two interfaces Let h3 be the new apparent depth of the needle
(air-glass and glass-air). It is easily seen from diagram that h h 13
µ ′ = 1 or h3 = 1 = = 7.65
r2 = i1 , i.e., the emergent ray is parallel to the incident ray – there h3 µ ′ 17.
is no deviation, but it undergoes a lateral shift.
here, h3 is less than h2 . Thus for focus the microscope
Medium hift should be moved up.
er al s
(Glass) Lat Distance by which microscope should be moved up
r2
= 8.5 − 7.65 = 0.85 cm
Medium i2 41. (c) The Sun is visible a little before the actual sunrise and
(Air) Medium until a little after the actual sunset due to refraction of light
r1 (Air)
i1 through the atmosphere.
42. (c) Time taken for 360° shift = 24 h
36. (a) When an object lying in a denser medium is observed Time taken for 1° shift = 24 / 360 h = 4 min
from rare medium, then real depth of object is more than 43. (c) Applying Snell’s law at air glass interface,
that observed depth. Normal
h h

y
37. (c) Apparent depth of the bottom of the beaker = 1 + 2

ra
In

ed
µ1 µ 2

c
id

ct
en

f le
i α

tr

Re
ay
Air
Glass
h2 (µ 2) Kerosene r Refracted

(µ 1) Water
h1
sin i a
= µg = µ (given)
sin r
∴ Apparent shift = Real depth − Apparent depth
⇒ sin i = µ sin r ⇒ i = sin −1 (µ sin r )
h h 
= ( h1 + h2 ) −  1 + 2  From laws of reflection, i = α = sin −1 (µ sin r )
 µ1 µ 2 
44. (c) In total internal reflection, light travels from an optically
 1  1
= h1 1 −  + h2 1 −  denser medium to a rarer medium at the interface, it is partly
 µ1   µ2 reflected back into the same medium and partly refracted
38. (b) As we know, refractive index of denser medium w.r.t. back to the second medium.
Real depth 45. (d) When the angle of incidence is more than a certain value
rare medium =
Apparent depth the angle of refraction becomes more than 90°. It result into
Real depth total internal reflection (critical angle).
39. (a) Refractive index, µ = 46. (d) For total internal reflection of light take place, following
Apparent depth
conditions must be obeyed.
(i) The ray must travel from denser to rarer medium.
a/2 Olive oil µ1
a
(ii) Angle of incidence (θ) must be greater than or equal to
Water µ2
critical angle (C ) i.e.,
a/2
µ 
C = sin −1  rarer 
 µ denser 
a a a 1 1
Apparent depth, d apparent = + =  + 
2µ 1 2µ 2 2  µ 1 µ 2 
Air
a  µ + µ 1  a (µ 1 + µ 2 ) Water
=  2 =
2  µ 1µ 2  2µ 1µ 2 C
Green
40. (a) Actual depth of the screw in water h1 = 13 cm
and apparent depth is h2 = 8.5 cm
h 13 1 b
µ water = 1 = = 1.53 Here, sinC = and n water = a + 2
h2 8.5 n water λ
When water replaced by a liquid of refractive index If frequency is less ⇒ λ is greater and hence, RI, n(water) is
µ ′ = 1.70 less and therefore, critical angle increases. So, they do not
then the actual depth remains same, but its apparent depth suffer reflection and come out at angle less than 90°.
changes.
sin i µ r
47. (c) Given, velocity of light in medium A = …(ii)
= v A = 2 × 10 ms 8 −1 sin r′ µ d
µ 
(µB) Also,critical angle = C = sin − 1  r 
µ d 
Rarer Medium B
Denser (µA) From Eqs. (i) and (ii),
C
Medium A  sin i 
C = sin − 1   [r′ = 90 − i] [sin ( 90 − i ) = cos i]
 sin r′ 
Velocity of light in medium B = vB = 2.5 × 108 ms − 1
 sin i 
vA µ B 2 4 ⇒ C = sin − 1  −1
 = sin (tan i )
Q = = =  cos i 
vB µ A 2.5 5
= sin − 1 (tan r ) (Q i = r )
⇒ B is rarer and A is denser medium.
µ  50. (b) For total internal reflection at AC , A > C
Since, critical angle for pair of media = C = sin − 1  r 
µ d 
( where µ r = refractive index for rarer medium and
µ d = refractive index for denser medium) A B
A
 4
∴ C = sin − 1  
 5 B
90°
48. (b) For glass-water interface, applying Snell’s law
sin i µ w C
= Where, C is critical angle
sin r µ g
 1
⇒ A > sin − 1  
90°  n
Air
1 1
r
⇒ sin A > ⇒ n > …(i)
r Water n sin A
Also, for total internal reflection at BC ,
Glass
 1 1
i B > sin − 1   ⇒ n > …(ii)
 n sin B
1 1
 µ sin r Since, A ≥ B ⇒ sin A ≥ sin B ⇒ ≤
⇒ µg =  w  …(i) sin A sin B
 sin i 
For total internal reflection at both surface i.e., for
For water air interface, Angle of incidence in water = r 1
Eqs. (i) and (ii) both to be true simultaneously, n >
sin r 1 1 sin B
Again, = ⇒ sin r = …(ii)
sin 90° µ w µw 1
n min =
 1 sin B
(µ w ) ×  
µ w  1 51. (c) For total internal reflection at surface AB,θ > C
From Eqs. (i) and (ii), µ g = ⇒ µg =
sin i sin i
 2
⇒ θ > sin − 1   …(i)
 2
49. (a) Since, reflected ray and refracted ray make an angle of
90° with each other. Also, for total internal reflection at surface CD,
Hence θ = 90°  3
so r + θ + r′ = 180° θ > sin − 1   …(ii)
 2
r + r′ = 90° or i + r′ = 90° (Q i = r) …(i)
 3  2
Applying Snell’s law, Since, sin − 1   > sin − 1   …(iii)
 2  2
y
In

ra
cid

ed

√2
en

ct
t

fle
ra

i r
Re
y

Denser (µd) θ θ
2
Rarer (µr) θ
r′ Refracted
√3
ray
For both the relations (i) and (ii) to be correct. Region I Region II Region III Region IV
 3
θ > sin − 1   r
 2 r
r
⇒ θ > 60° ⇒ θ min = 60° θ
52. (a) For total internal reflection at surface AC ,
µ1 µ2 µ3 µ4
θ>C
Applying Snell’s law at interface,
B A
µ 1 sin i = µ 2 sin r = µ 3 sin r ′ = constant
⇒ µ 1 sin i = µ 3 sin r ′ …(i)
θ n
Water Glass
⇒ n0 sin θ = 0 sin C
6
1 3 1  1
⇒ sin θ = × = ⇒ θ = sin − 1  
6 4 8  8
C
Note For refraction at multiple surface in sequence, we can apply
where, C = critical angle for glass water interface
Snell’s law directly for any two region as in Eq. (i).
µ  µ 
⇒ θ > sin − 1  r  ⇒ θ > sin − 1  water  56. (c) Due to total internal reflection of light.
µ
 d µ
 glass  57. (d) As critical angle ic is less for diamond, so light ray
 4 / 3 suffers more TIR’s before emerging and diamond shows a
⇒ θ > sin − 1   brilliance much more then glass.
 3/ 2
58. (a) Prisms are designed to bend light by 90° or by 180° by
 8
⇒ sin θ >   making use of total internal reflection [Fig. (a) and Fig. (b).
 9 Such a prism is also used to invert images without changing
1 their size [Fig. (c).
53. (a) If critical angle is θC then sin θC =
µ In the first two cases, the critical angle ic for the material of
the prism must be less than 45°.
Disc r Air

Water 45°
θc B B 45°
d 45°
A A

90°
90° 45°
A′
Light A′ B′ 45°
source (a) B′ (b)
1 r
so that tan θC = from diagram tan θC = B A′
(µ − 1)
2 1/ 2
d
A B′
r = d tan θC (r = radius of disc)
(c)
d  1 
r= 2 Q tan θC = 2 
(µ − 1)1 / 2  (µ − 1) 1/ 2 59. (c) Optical fibres are fabricated such that, light reflected at
one side of inner surface strikes the other at an angle larger
µ
54. (c) We know sin θ = 2 than the critical angle.
µ1 Even if the fibre is bent, light can easily travel along its length.
 c v Thus, an optical fibre can be used to act as an optical pipe.
As µ =  , sin θ =
 v v′ 60. (b) In optical fibres, core is surrounded by cladding, where
(here, v is speed in medium A and v′ is speed in medium B) the refractive index of the material of the core is higher than
v that of cladding to bound the light rays inside the core.
v′ =
sin θ 61. (c) Endoscope.
55. (b) Since, light beam just misses entering region IV, it 62. (d) Total internal reflection is the basis for following
must strike the region III region IV interface at critical phenomenon
angle (C ). (i) sparkling of diamond.
µ  n   3 (ii) optical fibre communication.
So, C = sin − 1  r  = sin − 1  0 / 8  = sin − 1  
µ
 d  0/ 6 
n  4 (iii) instrument used by doctors for endoscopy
63. (d) Figure shows the geometry of formation of image I of an u = 2R = − 30 cm
object O on the principal axis of a spherical surface with µ 1 = 1.5 ⇒ µ 2 = 1.0
centre of curvature C and radius of curvature R. The rays are
v=?
incident from a medium of refractive index n1 , to another of
µ 2 µ1 µ 2 − µ1
refractive index n2 . Applying, − =
v u R
N 1 1.5 (1 − 1.5)
n1 n2
i − =
v ( − 30 ) ( − 15 )
O r C I
1 1.5 − 0.5 1 0.5 1.5
M R ⇒ + = ⇒ = −
v 30 − 15 v 15 30
u v
⇒ v = − 60 cm
As before, we take the aperture (or the lateral size) of the The image is at a distance of 60 cm from surface ABC
surface to be small compared to other distances involved, so opposite to the direction of incident light.
that small angle approximation can be made. 69. (d) From first surface, µ = ∞
64. (a) As refraction formula for curved surface is µ1
n2 n1 n2 − n1
− =
v u R
65. (c) Here, u = − 100 cm, v = ?, R = + 20 cm, n1 = 1 C

and n2 = 1.5 µ2 =n
As, refraction formula for curved surface, we have v =2R
1.5 1 0.5 R
+ = ⇒ v = + 100 cm
v 100 20 v = + 2 R, where R is the radius of curvature.
The image is formed at a distance of 100 cm from the glass µ µ µ − µ1
Using equations 2 − 1 = 2
surface, in the direction of incident light. v u R
66. (a) The refractive index of the liquid must be equal to 1.47 (µ 1 = Refractive index of air)
in order to make the lens disappear. This means n1 = n2 . n µ 1 ( n − 1)
⇒ − = ⇒ n = 2.0
This given 1/ f = 0 or f → ∞. 2R ∞ R
67. (a) Applying, Lens’ maker formula, we get 70. (c) For refraction at surface AB,
µ 2 µ1 µ 2 − µ1
− = A
v u R
1.0 1.0

O O x
µ
Air µ Air
O
x O
R B
µ = − 2R
v=+∞ (µ 1 = µ ⇒ µ 2 = 1)
µ 1 µ −1 1 µ −1 µ µ µ − µ1
⇒ − = =+ = So, applying 2 − 1 = 2
∞ (− x ) + R x R v u R
1 µ 1− µ µ ( µ − 1)
 R  R ⇒ − = ⇒ =
∴ x=+ = ∞ − 2R −R 2R R
 µ − 1 (µ − 1)
⇒ µ = 2µ − 2
68. (b) Given, for refraction at spherical surface ABC, ∴ µ=2
A
µ2 71. (a) Given, u = − 8 cm
µ1
Views froe
m Converging lens, f = 10 cm
Mark this sid |v |
I Magnification, | m | = =?
O C B |u |
v=?
C
1 1 1 A diminished image by convex lens is obtained when object
Applying lens equation, − =
v u f is beyond 2 f , the image is between f and 2 f on other side.
uf ( − 8 ) × (10 ) So, OI = distance between object and screen = d
⇒ v= = = − 40 cm
u+ f ( − 8 ) + 10 ⇒ d > 2 f + 2 f ⇒ 4 f < 10
. m
40 1.0
So, |m | = =5 ⇒ f< ⇒ f < 0.25 m
8 4
|v | 74. (b) In general, spherical aberration is minimum when the
72. (c) Magnification, | m | =
|u | total deviation produced by the system is equally divided on
all refracting surfaces.
Case I u1 = − 0.15 m = −15 cm
A planoconvex lens is used for this purpose. In order that the
v1 = ? total deviation be equally divided on two surfaces, it is
m1 = ? essential that more parallel beam (or the incident and
Case II u 2 = − 0.2 m = − 20 cm refracted) be incident on the convex side.
v2 = ? Thus, when the object is far away from the lens, incident
rays will be more parallel than the refracted rays, therefore,
m2 = ? the object should face the convex side, but if the object is
Given, | m1 | = 2 | m2 | near the lens, the object should face the plane side.
| v1 | |v | | v1 | 2 × 15 3
⇒ =2 2 ⇒ = = This has been shown in figure.
| u1 | | u2 | | v2 | 20 2
3 3
⇒ | v1 | =
| v2 | or v1 = v2 O
I
2 2 O I
Applying lens equation,
1 1 1 1 1 1
− = or = − …(i) 75. (c) Spherical aberration is caused due to spherical nature of
v u f f v1 u1 lens. Paraxial and marginal rays are focused at different
1 1 1 places on the axis of the lens. Therefore, image so formed is
and = − …(ii)
f v2 u2 blurred. This aberration can be reduced by either stopping
paraxial rays or marginal rays, which can be done by using a
1 1 1 1
From Eqs. (i) and (ii), − = − circular annular mark over the lens.
v1 u1 v2 u2
 v2 − v1   u2 − u1 
⇒   = 
 v1 v2   u1 u2 
3
v2 −
v2
⇒ 2 =  − 5
 
3  300 Paraxial rays Marginal rays
v2 ⋅ v2
2 76. (d) As µ 2 > µ 1 , the upper half of the lens will become
− v2 − 5 − 300 diverging.
⇒ = or v2 = or v2 = + 20 cm
3 v22 300 15 As µ 1 > µ 3 , the lower half of the lens will become
Similarly, v1 = + 30 cm converging.
1 1 1 3 77. (a,d) Case 1
From Eq. (i), = − =
f + 30 ( − 15 ) 30
∴ f = 10 cm = 0.10 m I
73. (c) Image can be obtained on the screen if it is real. Real n
image of reduced size can be formed by a concave mirror or
a convex lens. 30 cm 60 cm
Using lens formula,
1 1 1 1 1 2
+ = ⇒ = + ⇒ f 1 = 20 cm
f I 60 30 f 1 f 1 60 60
O 2f f 2f
1  1 1 R
Further, = ( n − 1)  −  ⇒ f1 = = + 20 cm
f1  R ∞ n −1
d
Case 2 80. (d) From Lens Maker’s formula,
10 cm 1  1 1
= (1 µ 2 − 1)  − 
f  R1 R2 
n I

30 cm
µ3 µ2 µ1
Using mirror formula,
1 1 1 3 1 1 2
− = ⇒ − = =
10 30 f 2 30 30 f2 30
R For this concave lens to behave as diverging, f must be
f2 = 15 = ⇒ R = 30 ⇒ R = 30 cm negative.
2
 1 1
R
= +20 cm =
30
⇒ 2n − 2 = 3 ⇒ f 1 = + 20 cm Q  −  is negative
n −1 n −1  R1 R2 
Refractive index of lens is 2.5. So, (1 µ 2 − 1) > 0
Radius of curvature of convex surface is 30 cm. µ2
Image is erect and virtual. Focal length of lens is 20 cm. ⇒ > 1 or µ 2 > µ 1 …(i)
µ1
78. (b) When lens is in air,
If the lens is filled with L2 having refractive index
From Lens maker’s formula, [( n2 = µ 2 )] and placed in L1 having refractive index
1  1 1 [( n1 = µ 2 )].
= ( a µ g − 1)  −  …(i)
f1  R1 R2  81. (b) When lens is immersed in water,
When lens is immersed in liquid, 1  1 1
From Lens maker’s formula, = ( w µ g − 1)  − 
1  1 1 f2  R1 R2 
= ( l µ g − 1)  −  …(ii)
f2  R1 R2  1  1.5   1 1
⇒ = − 1  −  …(i)
On dividing Eq. (i) by Eq. (ii), we get f2  1.33   R1 R2 
f2 ( µ g − 1)
a
1  1 1
= l When lens is in air, = (1.5 − 1)  −  …(ii)
f1 ( µ g − 1) f1  R1 R2 
f2 (1.5 − 1)  l µg  On dividing Eq. (ii) by Eq. (i), we get
⇒ = Q µ g = 
f1  1.5   µl   0.5 × 1.33
 − 1 f2 =   × f1 = 4 f1 = 4 × 10 = 40 cm
 1.25   0.17 
( µ 2 = 125
. ) 82. (a) Since, lens is made of two layers of different refractive
f2 0.5 × 1.25 indices, for a given wavelength of light it will have two
⇒ = = 2.5 or f2 = 2.5 f1
f1 0.25 different focal lengths or will have two images at two
1
Hence, focal length increases by a factor of 2.5 when different points as ∝ (µ − 1) (from Lens maker’s
immersed in liquid. f
79. (a) Given, concave lens, µ g = 1.5 ; µ m = 1.75 formula).
83. (c) Given µ g = 15
.
Refractive index medium
µg 1.5
m
µg = =
µ m 1.75
Applying Lens maker’s formula,
1  1 1   1.5   1 1 
= ( m µ g − 1)  _ = − 1  −  Glass Glass
f  R1 R2   1.75   − R + R 
0.1
1 (1.5 − 1.75)  2 
⇒ = × −  µ oil = 17
f 1.75  R .
R = 20 cm
1.75 7
⇒ f = R = R = ( + 3.5 R ) From Lens Maker’s formula for the plano convex lens
0.25 × 2 2
1 1 1
Since, f is positive it behaves as convergent lens of focal = (µ − 1)  − 
length ( + 3.5 R ). f  R1 R2 
Here, R1 = R 1
For P<0 ⇒<0
and for plane surface, R2 = ∞ F
1 1  1 0.5 ⇒
1

d
<0 ⇒
1
<
d
∴ = (15
. − 1)  − 0 ⇒ =
flens R  flens R 10 400 10 400
When the intervening medium is filled with oil, then focal ∴ d > 40 cm
length of the concave lens formed by the oil 1
87. (b) Case I When lenses are in contact, P = = + 10 D
1  1 1 2 −14 . F1
= (17
. − 1)  − −  = − 0.7 × =
fconcave  R R  R R 1 1 1 1 1
= + ⇒ 10 = + …(i)
Here, we have two concave surfaces F1 f1 f2 f1 f2
1 1 1 0.5  −14
.  1 14. 0.4 Case II When lenses are d = 0.25 m apart
So, = 2× + = 2× +  = − =−
feq f f R  R  R R R 1 1 1 1 d
P= =+ 6D ⇒ = + − …(ii)
R 20 F2 F2 f1 f2 f1 f2
∴ feq = − =− = − 50 cm
0.4 0.4 From Eq. (i),
84. (c) At I, second focus of convex lens coincides with the first 0.25 0.25 1
6 = 10 − or f1 f2 = =
focus of concave lens. Hence, rays will become parallel to f1 f2 4 16
the optical axis after refraction from both the lenses. 1 1
⇒ × = 16 …(iii)
f1 f2
∴ P1 P2 = 16
I Hence, from Eq. (iii), only option (b) satisfies this relation.
Hence, P1 and P2 is 2 and 8.
100
0.75 m 0.25 m 88. (b) Power of lens (in dioptre) =
1.0 m focal length f (in cm)
85. (c) By Pythagoras theorem 100
f =
= 5 cm
32 + ( R − 3 mm )2 = R 2 20
⇒ 32 + R 2 − 2R ( 3 mm ) + ( 3 mm )2 = R 2 ⇒ R ≈ 15 cm 1  1 1
from Lens maker’s formula = (µ − 1) − 
c 3 × 108 f  R1 R2 
Refractive index of material of lens µ = =
v 2 × 108 for biconvex lens, R1 = + R
Here c = speed of light in vacuum R2 = − R
= 3 × 108 ms 2 1  1 1  2
cm = (µ − 1) +  = (µ − 1) 
R=
3 f  R R  R
v = speed of light in material of lens 3mm
= 2 × 108 ms 2 , µ = 3 / 2 1  2
= (µ − 1) 
5  5
From Lens Maker’s formula
1  1 1 2µ − 2 = 1 ⇒ 2µ = 3 ⇒ µ = ( 3/ 2 )
= (µ − 1) − 
f  R1 R2  89. (a) When the lenses are in contact, the power of the system is
P = P1 + P2 or P1 + P2 = 10 …(i)
Here, R1 = R and R2 = ∞ (for plane surface)
When lenses are separated by a distance
1 3  1
=  − 1   1
f  2   15 d = 0.25 m = m
4
⇒ f = 30 cm
The power is P = P1 + P2 − d P1 P2
86. (a) Given, P1 = P2 = + 5 D [1 D = 1m − 1 ] PP
⇒ P1 + P2 − 1 2 = 6 …(ii)
100 4
⇒ f1 = f2 = = + 20 cm
5 On solving Eqs. (i) and (ii), we get
For lenses separated by distance d, P P
1 1 1 d 1 1 1 d 10 − 1 2 = 6 ⇒ P1 P2 = 16
= + − ⇒ = + − 4
F f1 f2 f1 f2 F 20 20 400 So, P1 = 8 D and P2 = 2 D
1 1 d 1 1
⇒ = − ∴ f1 = m = 0.125 m ⇒ f2 = m = 0.5 m
F 10 400 8 2
90. (d) For equivalent focal length, Hence, the ray suffers minimum deviation and under
1 1 1 1 1 1 1 minimum deviation, the ray travels parallel to the base of the
= + + = + + prism inside the prism.
f f1 f2 f3 ( + 40 ) ( − 40 ) ( − 15 )
1 1 Hence θ = 0.
⇒ =− ⇒ F = − 15 cm  A + Dm 
f 15 sin  
 2 
1 100 20 94. (b) As, we know that µ =
Since, power i.e., P (in D) = =− =− A
F (in m) 15 3 sin
2
= − 6.67 D  A + Dm 
sin  
91. (c) Focal length of the combination A  2 
1 1 1 ⇒ cot =
= + …(i) 2 A
sin
f f1 f2 2
1  1 1  µ1 − 1 A sin  A + Dm 
We have = (µ 1 − 1) −  = cos  
f1  ∞ −R R 2 =  2 

1  1 1  − (µ 2 − 1) A A
and = (µ 2 − 1)  − = sin sin
f2  −R ∞ R 2 2
1 1  π A  A + Dm 
Putting these values of and in Eq. (i) sin  −  = sin  
f1 f2  2 2  2 
1 (µ 1 − 1) (µ 2 − 1) π A A Dm
= − ⇒ − = +
f R R 2 2 2 2
[µ − 1 − µ 2 + 1] µ 1 − µ 2 ⇒ Dm = π − 2 A ⇒ Dm = 180° − 2 A
= 1 =
R R 95. (b) Given, equilateral prism i.e., A = 60°
R
f = i=e=
3 3
A ⇒ = × 60° = 45°
µ1 −µ 2 4 4
1  1 1 From relation, A + D = i + e
92. (a) Using = (µ − 1)  − 
f  R1 R2  We have, 60° + D = 2 × 45°
here, R1 = 10 cm, R2 = − 10 cm, u = − 5 cm and µ = 133
. ⇒ D = 90° − 60° = 30°
1 1 1 96. (c) When light ray travels parallel to the base, the light
= (133
. − 1)  + 
f  10 10 suffers minimum deviations.
1 2 0.33 So, for minimum deviation, D min = 40°
= 0.33 × = ⇒ f = 1515
. cm i = e = 45° (from graph)
f 10 5
sin ( A + δ m ) / 2
uf − 5 × 1515
. 97. (d) Using µ =
Now, from lens formula, v = = A
u + f − 5 + 1515
. sin
2
− 75.75 π π
= = − 7.46 Here, A = = 45°, sin = = 45°
.
1015 4 4
⇒ v is −ve hence image will be formed on the same side. c = 3 × 10 ms
8 −1

93. (c) Let Dm be the angle of minimum deviation. Then, sin ( 45 + 45 ) / 2 0.707
∴ µ= ⇒ µ= = 1.85
 A + Dm  45 0.3826
sin   sin
 2  2
µ=
A c 3 × 108
sin
2 Therefore, v = = . × 108 ms−1
⇒ v = 162
µ 185
.
Given, A = 60° for equilateral prism and µ = 2
98. (a) Here, µ = 157
. , δ = 5.5°
 60° + Dm 
sin   using δ = (µ − 1) A
 2 
2= δ 5.5
 60°  A= = = 9.64 °
sin   µ − 1 157 . −1
 2 
99. (a) Consider a ray of light PQ incident an the surface AB
On solving, we get Dm = 30°
The given deviation, D = 30° and moves along RS , after passing through the prism ABC.
It is given that the incident ray A 102. (d) In vacuum, all colours have same speed which is equal to
suffers minimum deviation. the speed of light i.e., c = 3 × 108 ms −1 .
Therefore, the ray inside the 60°
prism must be parallel to the 45° 30° 45° 103. (b) For monochromatic light having single wavelength, only
base BC of the prism. P Q
30° 30° S deviation takes place and no dispersion occurs.
R
From the geometry of the prism 104. (b) When a white light is incident on a glass prism, the
and the ray diagram, it is clear that emergent light is seen to be consisting of several colours but
B C broadly the different component colours that appear in
angle of incidence, i = 45° sequence are violet, indigo, blue, green, yellow, orange and
angle of refraction r = r′ = 30° red (given by the acronym VIBGYOR).
angle of emergence, e = 45° 105. (b) Given, angle of prism = 5° ; µ r = 1.641, µ b = 1.659 g for
Therefore, minimum deviation suffered by the ray is this angle prism,
δ min = i + e − ( r + r′ ) = 90° − 60° = 30° D ≈ (µ − 1) A
Also we know that So, Db = (µ b − 1) A
 A + δm  ⇒ Dr = (µ r − 1) A
sin  
 2  Angle between the emergent blue and red rays
µ=
sin
A = Db − Dr = (µ b − µ r ) A
2 = (1.659 − 1.641) × 5
where, µ = refractive index of the material of the prism. = 0.018 × 5° = 0.09°
A = angle of prism = 60° 106. (b) The bending of red component of white light is least
 60° + 30°  while most for the violet component.
sin  
 2  sin 45° 1 / 2 2 107. (d) A lens can be viewed as a prism, it disperse light and
∴ µ= = = = = 2
60° sin 30° 1/ 2 2 image formed has coloured edges this is called chromatic
sin
2 aberration defect.
100. (a) δ = ( i1 + i2 ) − A ⇒ 40° = ( 35° + 79° ) − A 108. (d) Chromatic aberration occurs when a coloured image is
formed by a lens in white light.
⇒ A = 74 °
W
Now, we know that
 A + δm 
sin  
 2  V R
µ=
 A
sin  
 2
109. (d) The rainbow is an example of the dispersion of sunlight
If we take the given deviation as the minimum deviation, then by the water drops in the atmosphere. This is a phenomenon
 74 ° + 40°  due to combined effect of dispersion, refraction and
sin  
 2  reflection of sunlight by spherical water droplets of rain.
µ= = 151
.
 74 °  110. (c) The formation of primary rainbow occurs when red light
sin  
 2  from drop I and violet light from drop II reach the observers
The given deviation may or may not be the minimum eye. The violet light from drop II and red light from drop II
deviation. Rather it will be less than this value. Therefore, µ are directed at level above or below the observer. Thus the
will be less than 1.51. observer, say a rainbow with red colour on the top and violet
on the bottom.
Hence, maximum possible value of refractive index is 1.51.
Thus, the primary rainbow is a result of three step process,
101. (d) Angle of deviation first decreases and then increases that is, refraction, reflection and refraction as shown in figure.
with increase in angle of incidence.
t
ligh
Angle of deviation (δ)

Sun

1
50° Raindrops
2
40° 40°
42°
Observer
30° 111. (d) When light rays undergoes two internal reflections inside
20° 40° 60° 80°
Angle of incidence (i) a raindrop instead of one as in the primary rainbow, a
secondary rainbow is formed as shown in figure. It is due to
four step process. The intensity of light is reduced at the 121. (a) For distinct vision for human eye near point = D = 25 cm
second reflection and hence, the secondary rainbow is fainter Far point is at infinity.
than the primary rainbow. Further, the order of the colours is
reversed in it. 122. (c) He might be suffering from presbyopia.
123. (a) To rectify the short sighted defect, by using concave lens
Raindrops between the eye and the object, with the diverging effect
desired to get the image focused on the retina.
Light
124. (b) If the eye lens focusses the incoming light at a point
behind the retina, a convergent lens is needed to compensate
for the defect in vision. This defect is called farsightedness
50° 53°
or hypermetropia.
Observer
125. (c) Another common defect of vision is called astigmatism.
112. (d) The necessary condition for formation of rainbow is
This occurs when the cornea is not spherical in shape. If a
(i) Sun should be shining in part of the sky while it is person with such a defect in eye lens looks at a wire mesh or
raining in the opposite part of the sky. a grid of lines, focusing in either the vertical or the
(ii) The observer must stand with his back towards the Sun. horizontal plane may not be as sharp as in the other plane.
113. (a) The amount of scattering is inversely proportional of the
126. (a) The distance of normal vision is 25 cm. So, if a book is
fourth power of the wavelength. This is known as Rayleigh
at u = − 25 cm, its image should be formed at v = − 50 cm.
scattering.
Therefore, the desired focal length is given by
114. (a) At sunset or sunrise, the Sun’s rays have to pass through
1 1 1 1 1 1 1
a larger distance in the atmosphere. Most of the blue and = − ⇒ = − =
other shorter wavelengths are removed by scattering. The f v u f − 50 − 25 50
least scattered light reaching our eyes, therefore, the Sun ∴ f = + 50cm (convex lens)
looks reddish. This explains the reddish appearance of the 127. (d) The person should use a concave lens of focal length
Sun and full Moon near the horizon.
− 80
115. (a) We know, from Rayleigh’s scattering. = − 80 cm = m
100
1 − 100
Amount of scattering ∝ i.e., Power =
1
= = − 1.25 D
λ4 focal length 80
(Amount of scattering)1 λ 42
⇒ = 128. (a) Here, u = − 25 cm, v = − 75 cm
(Amount of scattering)2 λ 41
1 1 1
1  λ2
4
λ 2  1
1/ 4
1 = − i.e., f = 37.5 cm
⇒ =  ⇒ =  = f 25 75
4  λ1  λ 1  4 2 1 1
P= = = 2.67 D
∴ λ1 : λ 2 = 2 : 1 f 37.5
1 So, the corrective lens needs to have a converging power of
116. (b) From Rayleigh’s scattering, amount of scattering ∝
λ4 + 2.67 D.
Since, λ red is more, it is least scattered and hence, suitable 129. (b) The linear magnification m, for the image formed at the
for indication signals. near point d, by a simple microscope can be obtained by
1 using the relation.
117. (c) Amount of scattering ∝ 4 (Rayleigh’s law)
λ v  1 1 v
m = = v −  = 1 −
Thus, sky appears blue as blue light (short wavelength) is u v f f
scattered more than red and predominates. If red light will
Now, according to our sign convention, v is negative, and is
be scattered more, sky would appears red.
equal in magnitude to d. Thus, the magnification is
118. (b) Light enters the eye through a curved front surface, the  d
cornea. It passes through the pupil which is the central hole in m = 1 + 
the iris.  f

119. (d) The retina contains rods and cones which sense light Since, d is about 25 cm, to have magnification of six, one
intensity and colour, respectively, and transmit electrical needs a convex lens of focal length, f = 5 cm.
signals via the optic nerve to the brain which finally 130. (b) A single lens simple magnifier achieves is that it allows
processes this information. the object to be brought closer to the eye than D then the
120. (a) This property of the eye is called accommodation. magnification when the image is at infinity. In this case, we
will obtained the angular magnification.
131. (a) Fig. (i) describes myopia or short sightedness in which 138. (a) The first image is formed near the focal point of the
rays after refraction from lens meet (focussed) before the retina. eyepiece. The distance L, i.e., the distance between the
second focal point of the objective and the first focal point
132. (b) Hypermetropia is corrected by using convex lens.
of the eyepiece (focal length fe ) is called the tube length of
Focal length of lens used f = + (defected near point) the compound microscope.
f = + d = + 20 cm 139. (d)
100 100
∴ Power of lens = = = 5D
f (cm) + 20 L

133. (d) A simple magnifier or microscope is a converging lens of I


small focal length. In order to use such a lens as a microscope, fo+fe
the lens is held near the object, one focal length away or less,
and the eye is positioned close to the lens on the other side. We know, magnification of telescope,
The idea is to get an erect, magnified and virtual image of the f
m= o,
object at a distance so that it can viewed comfortably, i.e. at fe
25 cm or more.
fe −I
134. (d) A simple microscope has a limited maximum Here =
fe + u L
magnification (≤ 9 ) for realistic focal lengths.
fe −I
135. (d) For least distance of distinct vision, the angular ⇒ =
magnification of simple microscope is fe − ( fo + fe ) L
D  1
M = 1+ ⇒ M = 1 + DP ∴ Power ( P ) =  ⇒
fe I
= i.e., m =
L
f  t fo L I
D
and for normal adjustment M = ⇒ M = DP. 140. (c) Objective of a compound microscope is a convex lens.
f Convex lens forms real and enlarged image when an object
Hence, if the angular magnification of simple microscope is placed between focus and radius of curvature.
increases, then the power of the lens should increase. 141. (b) When final image formed at normal adjustment, then
136. (a) For magnification, a simple microscope may be used as length of compound microscope,
magnifying glass having converging lens. uo fo  D. fe 
L = vo + ue = + 
f = + 10 cm ( uo + fo )  D + fe 
⇒ u = − 9 cm
 1 1 1 1 1 1
uf − 9 × 10 Q = − and = − 
From lens formula, v = = = − 90  fo vo uo fe ue ve 
u + f − 9 + 10
v − 90 − 1.2 × 1 25 × 2.5
Magnification, m = = = 10 = + = 6 + 2.27
u −9 − 1.2 + 1 25 + 2.5
≈ 8.27 ≈ 8.3 cm
∴ Apparent area of the card through the magnifying
= 10 × 10 × 1 × 1= 100 mm2 = 1 cm2 142. (a) In modern microscopes, multicomponent lenses are used
for both the objective and the eyepiece to improve image
137. (b) Here, fo = 12
. cm, fe = 6.25 cm quality by minimising various optical aberrations (defects)
uo = − 2 cm, ve = − 25 cm in lenses.
1 1 1 1 1 1
For objective − = ⇒ = − 143. (d) Astronomical telescope depends on
vo uo fo vo 12
. 2 (i) light gathering power
⇒ vo = 3 cm (ii) resolution power
1 1 1 (iii) area of objective lens
For eyepiece − =
ve ue fe 144. (d) Telescope resolves and brings objects closer. Hence,
1 1 1 telescope with magnifying power of 20, the tree appears
⇒ − = 20 times nearer.
−25 ue 6.25
1 1 1 145. (a) Given, fo = F1 , fe = F2
− = +
ue 6.25 25 We know, angular magnification for telescope
ue = − 5 cm fo F F1
|M |= = 1 ⇒
Distance between two lenses = | vo | + | ue | = 3 + 5 = 8 cm fe F2 F2
146. (c) According to question, 152. (a) The main limitation of reflecting telescope is that the
Focal length of objective lens ( f0 ) = + 40 cm objective mirror focusses light inside the telescope tube.
Focal length of eyepiece lens ( fe ) = 4 cm 153. (b) The separation between the objective and therefore piece
Object distance for objective lens ( u 0 ) = − 200 cm = length of the telescope tube
Objective lens Eye-piece lens
⇒ f = fo + fe
here, fo = 150 cm = 1.5 m, fe = 5 cm = 0.05 m
4 cm f = 1.5 + 0.05 = 1.55 m

154. (a) The largest reflecting telescopes in the world are the part
200 cm v of keck telescopes in Hawaii.
155. (a) The largest telescope in India is in Kavalur, Tamilnadu
Applying lens formula for objective lens ( D = 2.34 m).
1 1 1 1 1 1 156. (c) Angle of incidence = Angle between incident ray and
− = ⇒ − =
v u f v −200 40 normal to the mirror = 0°
1 1 1 5−1 4
⇒ = − = = ⇒ v = 50 cm
v 40 200 200 200 Reflected ray
Image will be form at first focus of eyepiece lens.
Incident ray,
So, for normal adjustment distance between objectives and Normal
eye-piece lense (length of tube) will be
v + fe = 50 + 4 = 54 cm ⇒ Angle of reflection = 0° (from laws of reflection)
147. (a) For a telescope, Hence, the reflected ray retraces its path along the normal at
β f
Angular magnification = = o an angle 0° with normal.
α fe
157. (a) Refractive index of any pair of media is inversely
β 0.3
∴ = ⇒ β = 5° proportional to wavelength of light.
0.5° 0.03
Hence, λv < λr
| f |
148. (d) For telescope | m | = o = 5 …(i) ⇒ µv > µr
| fe |
where, λ v and λ r are the wavelengths of violet and red light
and length of the telescope and µ r and µ v are refractive index of violet and red light.
L = | fo | + | fe | = 36 …(ii)
158. (b) Optical fibre communication is based on the phenomenon
From Eqs. (i) and (ii),
of total internal reflection at core-clad interface.
⇒ fe = 6 cm and fo = 30 cm
The refractive index of the material of the cladding, hence,
149. (c) Given, power ( P1 ) = 40 D and power ( P2 ) = 20 D light striking at core-cladding interface gets totally internally
We have Peq = P1 + P2 = 40 D + 20 D = 60 D reflected. The light undergoes and reaches the other end of
1 1 1 the fibre.
So, = +
feq f1 f2 159. (a) Refractive index of diamond w.r.t. liquid
100 100 1 µ 6 1
⇒ f = = = 1.67 cm
l
µd = = d ⇒ =
Peq 60 sin C µ l 3 sin C
1
150. (d) For microscope, m =
L D
⇒ m∝
1 ⇒ sin C = = sin 45°
f0 fe f0 2
f0 ∴ C = 45°
For telescope, m= , m ∝ f0
fe 160. (d) In air or water a convex lens made of glass behaves as a
The magnifying power of microscope will decrease but the convergent lens but when it is placed in carbon disulfide, it
magnifying power telescope will increase. behaves as a divergent lens. Therefore, when a convergent
lens is placed inside a transparent medium having refractive
151. (d) Reflecting telescopes are used because of following index greater than that of material of lens, it behaves as a
advantages divergent lens.
(i) no chromatic aberration.
161. (c) When glass surface is made rough, then light incident on
(ii) parabolic reflecting surfaces are used.
it is scattered in different directions. Due to which its
(iii) Weighs of mirror are much less than a lens of equivalent
transparency decreases. There is no effect of roughness on
optical quality.
absorption of light.
162. (d) Magnification produced by mirror, 169. (c) Owl can move freely during night, because they have
I f f large number of cones on their retina which helps them to
m= = = . see in night.
O f− u x
f
where, x is distance from focus. 170. (c) The magnifying power of telescope is m = o . So, for
fe
Size of image ( I )
and m= high magnification, the focal length of objective lens should
Size of object (O )
be larger than eyepiece.
163. (a) For thin prisms, angle of prisms A is small d
Resolving power of a telescope = .
For small A, D min (minimum deviation) is also small. 1.22λ
 A + D min  For high resolving power, diameter ( d ) of objective should
sin  
 2  be higher.
So, µ= …(i)
sin ( A / 2 ) 171. (c) According to sign convention, all distances are measured
 A + D min  A + D min from the pole of the mirror or the optical centre of the lens.
sin   ≈ (Qsin θ ≈ θ for small θ)
 2  2 Object on left Mirror

A A Heights
and sin ≈ upwards
Incident light
2 2 positive
Using above approximations, X-axis
A + D min
µ= ⇒ D min = (µ − 1) A Distances against
2 incident light
Heights negative Distances along
A/2 incident light
downwards
negative positive
Hence, it can be seen that if A is small, D min is also small.
164. (d) Dispersion takes place because the refractive index of 172. (a) If n21 > 1, r < i, i.e. the refracted ray bends towards the
medium for different wavelengths (colours) is different. The normal. In such a case medium 2 is said to be optically
refractive index is inversely proportion to λ by Cauchy’s denser (or denser, in short) than medium 1.
expression as On the other hand, if n21 < 1, r > i, the refracted ray bends
b c
µ (λ ) = r + 2 + 4 away from the normal. This is the case when incident ray in
λ λ a denser medium refracts into a rarer medium.
Hence, deviation ( D ) = (µ − 1) A 173. (c) Speed of light depends on refractive index of medium.
Since λ red is more than other colours wavelength. The highest speed attainable in nature is c = 3 × 108 ms −1
So, deviation is least for red and have it appears farthest which is speed of light in vacuum. Thus speed of light in
from the base of the prism. water is less than speed of light in vacuum.

165. (a) In vacuum speed of light is independent of wavelength. 174. (c) It is possible that mass density of an optically denser
Hence, no dispersion takes places in vacuum. Thus, vacuum medium may be less than that of an optically rarer medium
is a non-dispersive medium in which all colours travel with (optical density is the ratio of the speed of light in two media).
the same speed. e.g., Turpentine and water. Mass density of turpentine is less
than of water but its optical density is higher.
166. (a) The rainbow is formed when the inner surface of the
µ 
water drop get internally reflected if the angle between the 175. (c) As, critical angle, C = sin − 1  r 
refracted ray and normal to the drop surface is greater than µ d 
the critical angle.  1  C
In this case, sin C =  Qµ = λ  …(i)
167. (a) I. Primary rainbow is a result of three-step process.  µd 
(i) Refraction at the first surface of raindrop.
Since, refractive index of material is inversely proportional
(ii) Total internal reflection from the second surface of
to the wavelength of the light. Wavelength of violet ( λ v ) is
raindrop.
least and hence, refractive index is maximum for violet.
(iii) Again refraction from the first surface of raindrop from Thus critical angle is minimum from Eq. (i).
where the light finally emerges out. The intensity of light
is reduced at the second reflection and hence, the 176. (d) When white light pass through a prism, bending of red
secondary rainbow is fainter than the primary rainbow. component of white light is least while it is most for the
violet. Equivalently, red light travels faster than violet light
168. (c) The light from a distant object arriving at the eye lens in a glass prism.
may get converged at a point in front of the retina. This type
of defect is called nearsightedness or myopia. This means 177. (b)
that the eye is producing too much convergence in the I. At sunset or sunrise, the Sun’s rays have to pass through
incident beam. a larger distance in the atmosphere. Most of blue and
other shorter wavelengths are removed by scattering. sin i
Applying Snell’s law, 1 = air
µ ice
The least scattered light reaches our eyes, therefore Sun sin r1
looks reddish. i1=
60°
II. In secondary rainbow light undergoes two internal Air
reflections inside a raindrop and hence the order of A Ice
r1
colour gets reversed. r1
Ice
178. (c) From Lens maker’s formula,
B Glass
1  µ lens  1 1 r2
In air = − 1  − 
f1  µ air   R1 R2 
1  µ lens  1 1 sin 60° µ ice
In water = − 1  − ⇒ = …(i)
f2  µ water   R1 R2  sin r1 1
µ lens ( 3/ 2 ) µ lens ( 3/ 2 ) sin 60° 3/ 2 3 3
Q < ⇒ sin r1 = = = … (ii)
µ water ( 4 / 3 ) µ air (1) µ ice 4/ 3 8
1 At B, Ice - glass interface
⇒ or power is less in water but f2 is positive, hence it Applying Snell’s law,
f2
sin r1 ice µ glass
behaves as convex lens and not as concave lens. = µ glass = …(A)
sin r2 µ ice
179. (c) A lens with large focal length have less power.
1 On multiplying Eqs. (i) and (ii), we get
i.e., P∝
f sin 60° sin r1 µ ice µ glass sin 60°
× = × ⇒ = µ glass
1 sin r1 sin r2 1 µ ice sin r2
or f ∝
P sin 60° ( 3 / 2 ) 3/ 2  1 
⇒ sin r2 = = = =  …(B)
180. (a) Astigmatism can be corrected by using cylindrical lens of µ glass 1.5 3/ 2  3
desired radius of curvature with an appropriate directed axis.
Also, refractive index of ice w.r.t. glass
f
181. (a) For astronomical telescope, | m | = o . air
µ 4/ 3 8
fe ⇒ glass
µ ice = air ice = = …(C)
µ glass 3/ 2 9
So, magnification is inversely proportional to focal length of
the eyepiece and directly proportional to focal length of 185. (d) A → 2, B → 3, C → 1 is correct option. If the object is at
objective. The larger fo and smaller fe will given better a distance f , the image is at infinity. However, if the object
magnification. is at a distance sightly less than the focal length of the lens,
the image is virtual and closer than infinity. Although the
182. (a) The largest lens objective in use has the diameter 40 inch
closest comfortable distance for viewing the image is when
(~ 1.02 m). It is at the Yerkes observatory in Wisconsin,
it is at the near point (distance D ≅ 25 cm), it causes some
USA. Such big lenses tend to be very heavy and therefore,
strain on the eye. Therefore, the image formed at infinity is
difficult to make and support by their edges. Further, it is
often considered most suitable for viewing by the relaxed eye.
rather difficult and expensive to make such large sized
lenses which form images that are free from any kind for 186. (c) In case of concave mirror or convex lens (A), (C) and
chromatic aberration and distortions. (D) image can be real, virtual, diminished, magnified or of
183. (c) From laws of reflection, same size. In case of convex mirror (B), image is always
Normal virtual (for real objects).
y
ra

187. (a)
ed
In

ct

(A) → Since µ 1 < µ 2 , the ray of light will bend towards


c
id

f le

α
en

α
Re

normal after first refraction.


t

β
ra
y

(B) → µ 1 > µ 2 , the ray of light will bend away from the
φ Angle of
α deviation
normal after first refraction.
(C) → since µ 2 = µ 3 means in second refraction there will
be no change in the path of ray of light.
(A) Angle of incidence = Angle of reflection = α
(D) → Since µ 2 > µ 3 , ray of light will bend away from the
(B) The value of angle β = π / 2 − α normal after second refraction.
(C) Angle of deviation = φ = π − 2α (From figure)
Therefore the correct options are as under
184. (b) In this case, the light gets refracted twice from air to ice (A) → 1, 3 (B) → 2, 4, 5
at A and from ice to glass at B. We have find angle r2 . (C) → 1, 3, 5 (D) → 2, 4
At A, Air - ice interface
188. (c) Minimum Deviation On multiplying Eqs. (i) and (iii), we get
The angle of minimum deviation occurs when i = e and sin i µ 3
= …(iv)
r1 = r2 and is given by sin e µ 4
( A + D min )
sin Since, µ 3 ≠ µ1 ⇒ sin i ≠ sin e or i≠e
  A 
µ= 2 ⇒ D min = 2 sin −1 sin   × µ  − A 193. (d) From relation (iv), in above answer.
A   2  
sin sin i µ 3
2 =
Substituting µ = 1.5 and A = 60° sin e µ 4
D min = 2 sin −1 (0.75) − 60° = 37° …(A) Q µ 3 = µ1 ⇒ sin i = sin e
∴ i=e
Hence, the angle of minimum deviation is 37°.
Maximum Deviation …(B) and (C) Also, r = r1 (alternate angle, from figure)
The deviation is maximum when i = 90° or e = 90° that is at 194. (b) At angle of incidence iC i.e., critical angle for pair of
grazing incidence or grazing emergence. media in the denser medium corresponding to which angle
A of refraction in the rarer medium is 90°.
195. (d) So, for i > ic , total internal reflection takes place. Hence,
the ray gets reflected into the denser medium.
i=90° e
r1 r2

Rarer
Denser ay 180°–2i
r i
ent i > ic
id
Let i = 90° Inc Reflected ray
So, r1 must be at critical angle for pair (air glass) interface.
µ   1 From figure, angle of deviation = (180° − 2i ) clockwise
⇒ r1 = c = sin −1  r  = sin −1   = (180° + 2i ) anti-clockwise
µ d  µ 
µ 
 2 196. (b) As, critical angle = iC = sin − 1  r 
⇒ r1 = sin −1   = 42° …(C) µ d 
 3
 1.0  2
Also, r1 + r2 = A ⇒ r2 = A − r1 = 60° − 42° = 18° = sin − 1   = sin − 1  
sin r2 1  1.5  3
Using = ⇒ sin e = µ sin r2 = 1.5 sin 18°
sin e µ 197. (a) For total internal reflection, the light must pass from the
denser to rarer medium.
⇒ sin e = 0.463 ⇒ e = 28°
Q µ A > µ B , so material A is denser than material B.
Deviation = D max = i + e − A = 90°+28°−60° = 58° …(B)
So, light must travel from A to B.
Hence, the maximum deviation for the prism is 58°.
198. (a) For pair of media A and B, critical angle = C (let)
189. (d) A concave mirror forms real and virtual images, whose
µ Refractive index of rarer medium
magnification can be negative or positive depending upon ⇒ sin C = r =
the position of the object. If object is placed between focus µ d Refractive index of denser medium
and pole the image obtained will be virtual and its µ B 1/sin θ B
magnification will be positive. In all other cases, concave = =
µ A 1/sin θ A
mirror forms real images whose magnification will be
negative. A convex mirror always forms a virtual image  1 1
Qsin θ A = and sin θ B = 
whose magnification will always be positive.  µA µ B 
192. (d) Applying Snell’s law at interface AB sin θ A  sin θ A 
∴ sin C = ⇒ C = sin − 1  
sin i µ 2 sin θ B  sin θ B 
= …(i)
sin r µ 1
199. (c) Given, µ = 1.5
Again, applying Snell’s law at interface DC A
sin r1 µ 3
= …(ii)
sin e µ 2
P
i=60°
From figure, r = r1 (alternate angles) r1
Q
e
r2
sin r µ 3
⇒ = …(iii)
sin e µ 2
Angle of incidence = i = 60° 208. (a, c) Normal is perpendicular to the tangent to surface at the
Refracting angle of prism = A = 60° point of incidence i.e., the normal is along the radius, the
Angle of emergence = e = ? line joining the centre of curvature of the mirror to the point
Angle of deviation = D = ? of incidence.
Applying Snell’s law at P, 209. (a, b, d) The ratio of the sine of the angle of incidence to the
sin 60° 1.5 3 2 1 sine of angle of refraction is constant.
= ⇒ sin r1 = × = sin i
sin r1 1 2 3 3 hence = n21 …(i)
sin r
 −1  1  
⇒ r1 ≈ 35° 16 ′ sin   = 35° 16 ′ where, n21 is a constant, called the refractive index of the
 3  medium 2 w.r.t. the medium 1. Eq. (i) is the well known
Using r1 + r2 = A , we get, Snell’s law of refraction and on depends on temperature.
r2 = 60° − 35°16 ′ = 24 ° 44 ′ 210. (a, b, d) Refractive index depends on optical densities of
sin r2 1 mediums 1 and 2, it also depends on temperature (or optical
At point Q, = density varies with temperature), but it is independent of
sin e 1.5
angle of incidence. Refractive index for a pair of medium is
⇒ sin e = 1.5 sin 24 ° 44 ′ (sin 24 ° 44 ′ = 0.42)
same for all angle of incidence.
⇒ sin e = 0.63 ⇒ e = 39°
211. (a, b, d) If n21 is the refractive index of medium 2 with
200. (a)Q A+D=i+e
respect to medium 1 and n12 the refractive index of medium
⇒ D = i + e − A = 60° + 39° − 60° 1 with respect to medium 2, then it should be clear that
(Using, e = 39° from above solution) 1
= 39° n12 =
n21
201. (a) From the principle of reversibility of path it can be It also follows that if n32 is the refractive index of medium
verified that, if i and e are interchanged, deviation remains 3 with respect to medium 2, then n32 = n31 × n12 , where, n31
same. Hence, same deviation is obtained for angle of is the refractive index of medium III with respect to medium I.
incidence 60° and 39°. n
n21 = 2 a
202. (d) The eyepiece lens of an astronomical telescope should n1 a
have least possible focal length and small aperture. Hence,
212. (a,b) I. Convex lens is a converging lens provided refractive
we shall use lens L4 as the eyepiece whose focal length
1 index of the material of the lens is greater than the
= m = 5 cm and aperture is 0.02 m. surrounding medium in which the lens is kept.
20
1 1 1
203. (a) The objective lens of an astronomical telescope should From Lens makers formula, = (µ 2 − 1)  − 
have highest possible focal length as well as aperture. Hence, f  R1 R2 
we shall select L1 of power 1 D (or focal length 1m) and Refracting index of lens varies inversely with the
aperture 0.1 m. wavelength of light used.
204. (d) Here, fo = 20 m and fe = 2 cm = 0.02 m. II. For virtual object, image is real for convex lens.
In normal adjustment, length of telescope take
L = fo + fe = 20.02 m
f 20
Magnification = o = = 1000 I
fe 0. 02 O
(Virtual object)
The image formed in inverted with respect to the object. Real image
1 1 1 4
205. (c) Critical angle i.e., µ = = = = .
sin C sin 48.6 0.75 3 III. For object (real) between and O, image is
virtual.
206. (d) Light cannot undergoes total internal reflection when it is
travelling from air to water, i. e.,from rarer to denser medium. 213. (a, d) Final image is formed at infinity if the combined focal
207. (a) From total internal reflection of light, length of the two lenses (in contact) becomes 30 cm.
1 1 1
As we know that, µ =
1
⇒ sin C =
1 Thus, from Lens maker’s formula, = +
sin C µ 30 20 f
2 i.e. when another concave lens of focal length 60 cm is kept
As, sin C = 1 / ( 3/ 2 ) = = 0.6667 in contact with the first lens. Similarly, let µ be the
3 refractive index of a liquid in which focal length of the
C = sin −1 (0.6667) = 41.8° given lens becomes 30 cm. Then,
1 3   1 1 1 1 1 4+5 9
=  − 1  −  …(i) ⇒ = + = =
20  2   R1 R2  v 15 12 60 60
Distance of image from the mirror v = 6.7 cm
1  3/ 2   1 1
= − 1  −  …(ii) The positive sign shows that the image is formed behind the
30  µ   R1 R2  mirror. Using the formula of magnification.
9 v I − 6.7 I
From Eqs. (i) and (ii), µ = m= − = ⇒ =
8 u O − 12 4.5
214. (a, b) In vacuum, the speed of light is independent of Size of images I = 2.5 cm
wavelength. Thus, vacuum (or air approximately) is a As I is positive, so image is erect and virtual
non-dispersive medium in which all colours travel with the Magnification m is given by
same speed. I 2.5 25 5
m= = = =
215. (a, b, c) In compound microscope objective lens forms a O 4.5 45 9
real, inverted, magnified image of the object. This serves as
the object for the second lens, the eyepiece, which functions 218. (c) When tank is filled with water:
essentially like a simple microscope or magnifier, produces Real depth = 12.5 cm
the final image, which is enlarged and virtual. The first Apparent depth = 9.4 cm
inverted image is thus meet (at or within) the focal plane of Refractive index of water is
the eyepiece, at a distance appropriate for final image Real depth
a
µw = = 1.32
formation at infinity, or a little closer for image formation at Apparent depth
the near point.
When tank is filled with liquid
Clearly, the final image is inverted with respect to the
original object. Real depth = 12.5 cm
Real depth
216. (b) Here, h1 = 2.5 cm, u = − 27 cm, R = −36 cm Refractive index of liquid =
R Apparent depth
f = = −18 cm 12.5
2 or 1.63 =
(Q R is − ve for a concave mirror) Apparent depth
By mirror formula, 12.5
1 1 1 1 1 −3 + 2 1 Apparent depth with liquid = cm = 7.669 cm ~ 7.7 cm
= − = + = =− 163
.
v f u −18 27 54 54 Distance through which the microscope has to be moved
or v = −54 cm = 9.4 − 7.7 = 1.7 cm
Thus the screen should be placed at 54 cm from the mirror 219. (b) Use relation between critical angle and refractive index.
on the same side as the object. When light is incident at an angle of incidence equal to the
Magnification, critical angle. The angle of refraction is 90° and light will
h v −54 pass through the interface of the two media.
m= 2 = − = − = −2
h1 u −27
Size of image, C A B
h2 = −2 × 2.5 = − 5 cm
Negative sign shows that the image is real and inverted.
217. (a) Given, focal length of convex mirror f = + 15 cm (Focal ic
ic ic
ic
length of convex mirror is taken as positive)
Distance of object u = − 12 cm
Size of object O = 4.5 cm O
Let the bulb is placed at point O
AB = AC = r
If the light falls at an angle of incidence equal to critical
F C angle iC , then only a circular area is formed because if angle
of incidence is less than the critical angle it will refract into
air and when angle of incidence is greater than critical angle
then it will be reflected back in water.
Using the mirror formula,
The source of light is 80 cm below the surface of water i.e.,
1 1 1 1 1 1 AO = 80 cm, µ w = 133
= + ⇒ = − .
f v u 15 v 12
Using the formula for critical angle, 222. (a) Here, the point P is on the right side of lens acts as
1 1 vertical object.
sin iC = ⇒ sin iC = = 0.75
µw 133
. Given, distance of object from the lens u = 12 cm
iC = 48.6°
AB r 12 cm
In ∆OAB tan iC = or tan ic =
AO l P
r = l tan iC = 80 tan 48.6
O
r = 80 × 11345
. = 90.7 cm
Area of circular surface of water, through which light will Focal length of convex lens f = + 20 cm
emerge 1 1 1 1 1 1
A = π r2 Using Lens formula, − = ⇒ − =
v u f v 12 20
A = 314
. × ( 90.7 )2 = 25865.36 cm 2 1 1 1 3+ 5 8
= + = =
A = 2.58 m2 v 20 12 60 60
220. (a) When the prism is placed in air v = 7.5 cm
δ m = 40°, A = 60° Thus, the beam converges on the right side of lens at a
distance of 7.5 cm.
∴ Refractive index of the prism material is
A + δm 60°+40° 223. (d) Given, concave lens, f = − 21cm, u = − 14 cm
sin sin
2 2 sin 50° 0.7660 Object size O = 3 cm
a
µg = = = = = 1532
.
A 60° sin 30° 0.5000 Applying lens formula,
sin sin
2 2 uf ( − 14 ) × ( − 21) − 42
v= = = cm = − 8.4 cm
When the prism is placed in water u + f ( − 14 ) + ( − 21) 5
A + δm
′ 60° + δ ′m Using the formula of magnification of lens
sin a
µ g sin
ω 2 2 v I
µg = or a = m = = ,where I = height of image
A µ ω
60 ° u O
sin sin
2 2 ( − 8.4) I
60°+ δ ′m ⇒ = ⇒ I = 1.8 cm
sin − 1.4 +3
1.532
or = 2 v − 8.4
1.33 sin 30° Magnification of a lens, M = = = 0.6
u −1.4
60° + δ m
′ 1.532
or sin = × 0.5 = 0.5759 224. (d) Since, v is negative and I is positive, the image is virtual
2 1.33
and erect and at a distance of 8.4 cm from the lens on the same
δ′
∴ 30°+ m = sin −1 (0.5759) = 35°10' or δ m ′ = 10°20' side as object and height of the image is 1.8 cm and hence,
2 diminished.
221. (c) Given, the refractive index of glass with respect to air If the object is moved further away from the lens between O
a
µ g = 155
. (Q both faces have same radius of curvature) and F, the image moves towards the lens (never beyond
focus). The size of image decreases gradually.
For double convex lenses R1 = R , R2 = − R
225. (a) Given, f1 = + 20 cm, f2 = + 30 cm
(For double convex lens, one radius is taken positive and
other negative) So, for equivalent focal length
Focal length of lens, f = + 20 cm 1
=
1
+
1

1
=
1
+
1
f f1 f2 f 20 30
20 × 30 600
∴ f = = = 12 cm
R1 R2 20 + 30 50
226. (b) Here, f0 = 0.8 cm, u0 = − 0.9 cm, v0 = ?
1 1 1
Using the Lens maker’s formula As − =
v0 u0 f0
1  1 1 1  1 1
= ( a µ g − 1) −  ⇒ = (155
. − 1) +  1 1 1 1 1 0.9 − 0.8 0.1
f  R1 R2  20  R R ∴ = + = − = =
v0 f0 u0 0.8 0.9 0.9 × 0.8 0.8 × 0.9
1 2
= 0.55 × ⇒ R = 0.55 × 2 × 20 = 22 cm 0.8 × 0.9
20 R or v0 = = 7.2 cm
0.1
Thus, the required radius of curvature is 22 cm.
Now for the eye-piece, we have 231. (b) The minimum distance between an object and its real
fe = 2.5 cm, ve = − D = −25 cm, ue = ? image is 4 f .
1 1 1 1 1 −1 − 10 −11 D 3m
∴ = − =− − = = ∴ 4 f max = D or f max = = = 0.75 m
ue ve fe 25 2.5 25 25 4 4
25 232. (d) As shown in figure, let O and I be the positions of object
or ue = − = − 2.27 cm
11 and image respectively and L1 and L2 be the two conjugate
Hence the separation between the to lenses positions of the lens.
L1 L2
= v0 + | ue | = 7.2 + 2.27 = 9.47 cm
Magnifying power,
v  D  7.2  25  O I
m = m0 × me = 0 1 +  = 1 +  = 88
| v0 |  fe  0.9  2.5
x 20 cm x
227. (a) Given, focal length of objective lens fo = 144 cm.
Obviously, x + 20 + x = 90 cm or x = 35 cm
Focal length of eye-piece fe = 6 cm When the lens is in position L1 , we have
Magnifying power of the telescope in normal adjustment u = − x = −35 cm, v = 20 + x = 20 + 35 = 55 cm
(i.e., when the final image is formed at ∞) 1 1 1 1 1 7 + 11 18
f 144 ∴ = − = + = =
m= − o = − = − 24 f v u 55 35 385 385
fe 6 385
or f = = 21.4 cm
∴ Separation between lenses L = fo + fe = 144 + 6 = 150 18
cm
233. (b) Angle of prism, A = 60°
228. (d) Here, f0 = 15 cm, fe = 1.0 cm = 0.01 m Refractive index of prism µ = 1.524
f 15
Angular magnification, m = 0 = = 1500 Let i be the angle of incidence. The critical angle is ic
fe 0.01 because it just suffers total internal refraction, so we use
critical angle,
229. (c) Let d be the diameter of the image in metres. Then angle
1 1
subtended by the moon will be sin iC = = = 0.6561 ⇒ iC = 41°
µ 1524.
Diameter of moon 3.48 × 106
α= =
Radius of lunar orbit 3.48 × 108 A

Angle subtended by the image formed by the objective will 60°


also be equal to α and is given by
Diameter of image of moon d i1
α= = r1 ic
f0 15
d 3.48 × 106
∴ = B C
15 3.8 × 108
Diameter of image of moon, For a prism r1 + r2 = A here r2 = ic
3.48 × 106 × 15 ∴ r1 + iC = A ⇒ r1 + 41° = 60° ⇒ r1 = 19°
d= sin i1
3.8 × 108 Using the formula, µ =
sin r1
3.48 × 15 × 10–2
= = 13.73 cm or sin i1 = 1524
. sin 19° = 1524
. × 0.3256
3.8
or i1 = sin −1 ( 0.4962 )
230. (b) Given, thickness of glass slab (real depth) = 15 cm
⇒ i1 = 29° 75′
Refractive index of glass a µ g with respect to air = 15
.
Thus, the angle should be 29°75′.
Real depth 15
Using the formula a µ g = = 234. (a) To see objects at infinity, the eye uses its least
Apparent depth a µ g converging power = 40 + 20 = 60 dioptres
15 ∴ Approximate distance between the retina and the cornea
Apparent depth of pin y = = 10 cm eyelens
1.5
Distance by which the pin appears to be raised = focal length of the eyelens
= Real depth − Apparent depth= 15 − 10 = 5 cm 100 100 5
= = = cm
The answer does not depend on the location of the slab. P 60 3
To focus an object at the near point on the retina, we have 239. (b) (i) Here, area of each square (or object) = 1 mm2
5
u = −25 cm, v = cm u = − 9 cm, f = +10 cm
3
1 1 1
∴ Focal length f should be given by As − =
v u f
1 1 1 3 1 15 + 1 16 25
= − = + = = ⇒ f = cm 1 1 1 1 1 9 − 10 1
f v u 5 25 25 25 16 ∴ = + = − = =−
v f u 10 9 90 90
Corresponding converging power = 64 D.
Power of the eye-lens = 64 − 40 = 24 D or v = − 90 cm
Thus the range of accommodation of the eyelens is roughly Magnitude of magnification is
20 to 24 D. v 90
m= = = 10
235. (b) Initially power of spectacles = − 1D, i.e., focal length is |u | 9
( − 100 ) cm. It means that far point of the person is 100 cm Area of each square in the virtual image
and near point is normal i.e. 25 cm. Due to the old age the = (10 )2 × 1 = 100 mm2 = 1cm2
person use + 2 D spectacles i.e., focal length f = 50 cm
D 25
So, u = − 25 cm and f = 50 cm (ii) Magnifying power, M = = = 2.8
|u | 9
1 1 1
From the lens formula, = + 240. (a) We assume the microscope in common usage, i.e., the
50 v 25
1 1 1 1− 2 1 final image is formed at the least distance of distinct vision,
⇒ = − = =− ⇒ v = −50cm D = 25 cm, fe = 5 cm
v 50 25 50 50
The near point is a 50 cm. ∴ Angular magnification of the eye piece is
D 25
236. (c) This defect is called astigmatism. It arises because the me = 1 + = 1+ =6
fe 5
curvature of the cornea plus eyelens refracting system is not
the same in different planes. The eyelens is usually As total magnification, m = me × m0
spherical, ∴ Angular magnification of the objective is
i.e., has the same curvature in different planes but the cornea m 30
is not spherical in case of an astigmatic eye. The defect can m0 = = =5
me 6
be corrected by using a cylindrical lens with its axis along
the vertical. As real image is formed by the objective, therefore,
237. (a) For the closest distance, v = −25 cm, f = 5 cm, u = ? v
m0 = 0 = −5
v0
1 1 1
As − = or v0 = − 5 u0 ⇒ f0 = 1.25 cm
v u f
1 1 1
1 1 1 1 1 −1 − 5 −6 Now, − =
∴ = − = − = = v0 u0 f0
u v f −25 5 25 25
1 1 1 −6 1
25 or − = or =
or u=− cm = − 4.2 cm −5u0 u0 125
. 5u0 125
.
6
6 × 1.25
This is the closest distance at which the man can read the book. or u0 = − = − 1.5 cm
For the farthest image 5
v = ∞, f = 5 cm, u = ? Thus, the object should be held at 1.5 cm in front of the
objective lens.
1 1 1 1 1 1 1
∴ = − = − = 0− = − Also v0 = −5 u0 = − 5 × ( −1.5 ) = 7.5 cm
u v f ∞ 5 5 5
1 1 1
u = −5 cm As − =
ve ue fe
This is the farthest distance at which the man can read the
1 1 1 1 1
book. ∴ = − = − (Q ve = − D = −25cm )
ue ve fe − 25 5
238. (c) Maximum angular magnification is
−1 − 5 6
d 25 = =−
= =6 25 25
u min 25 / 6
−25
Minimum angular magnification is ue = = − 4.17 cm
6
D 25
= =5 ∴ Separation between the objective and the eyepiece
u max 5
= | ue | + | v0 | = 4.17 + 7.5 = 11.67 cm
241. (d) Here, f0 = 140 cm and fe = 5 cm Distance measured in second case
(i) In normal adjustment, magnifying power, = Focal length of the convex lens
f 140 i.e., f1 = + 30 cm
m= 0 = = 28 The focal length f2 of the plano-concave lens is given by
fe 5
1 1 1 1 1 1
(ii) When the final image is formed at the least distance of + = or = −
f1 f2 F f2 F f1
distinct vision (25 cm)
f  f  1 1 2− 3 1
m = 0 1 + e  = − ⇒ = =−
fe  D 45 30 90 90
∴ f2 = − 90 cm
140  5
= 1 +  = 28 × 1.2 = 33.6 Now for the equiconvex lens, we have
5  25
R1 = R , R2 = − R , f = 30 cm, µ = 1.5
242. (c) (i) Angle subtended by the 100 m tall tower at 3 km Using lens maker’s formula
away is
1 1 1 1  1 1
100 1 = ( µ − 1)  −  or = (1.5 − 1)  + 
α ≈ tan α = = rad f  R1 R2  30 R R
3 × 103 30
1 2
Let h be the height of the image of tower formed by the = 0.5 ×
objective. Then angle subtended by the image produced 30 R
by the objective will also be equal to α and is given by or R = 0.5 × 2 × 30 cm = 30 cm
α=
h
=
h For plano-convex lens, f = −90 cm
f0 140 For concave surface, R1 = − R = −30 cm
h 1 140 14 For plane surface, R2 = ∞
∴ = or h = = = 4.67 cm
140 30 30 3 1 1 1
As = (µ − 1)  − 
(ii) Magnification produced by the eyepiece is  1
f R R 2
D 25
me = 1 + = 1+ =6 1  1 1 −30 1
fe 5 ∴ = (µ − 1)  −  or µ − 1 = =+
− 90  −30 ∞  −90 3
14
∴ Height of the final image = h × me = × 6 = 28 cm 1
3 or µ = 1 + = 1.33
3
243. (a) Given, distance between objective mirror and another
245. (a) A
mirror d = 20 mm
Radius of curvature of objective mirror = R1 = 220 mm
220
∴ Focal length of objective mirror, f1 = = 110 mm
2 i=θ
Radius of curvature of small mirror = R2 = 140 mm
140 B C
∴ Focal length of small mirror, f2 = = 70 mm
2 Given, i = θ ; A = 5° ; µ = 1.5
The image of an object placed at infinity, formed by the For ray emerging normally from AC, e = 0
objective mirror will act as a virtual object for small mirror. From relation, A + D = i + e, we have
So, the object distance for small mirror u = f1 − d ⇒ 5° + D = θ + 0 ⇒ i = θ = 5° + D
i.e., u = 110 − 20 = 90 mm Also, for small angles, D = (µ − 1) A = 0.5 x = 2.5°
Using mirror formula So, i = θ = 5° + 2.5° = 7.5°
1 1 1 1 1 9− 7 2 246. (d) Since, v ∝ λ , the light of red colour is of highest
= − = − = =
v f2 u 70 90 630 630 wavelength and therefore of highest speed. Therefore, after
v = 315 mm or v = 315 . cm travelling through the slab, the red colour emerge first.
Thus, the final image is formed at 315 mm away from small 247. (c) When an object approaches a convergent lens from the left
mirror. of the lens with a uniform speed of 5 ms −1 , the image moves
244. (d) Distance of the needle from the lens in the first case away from the lens with a non-uniform acceleration.
= Focal length F of the cmbination of the convex lens and 248. (b) A passenger in an aeroplane may see a primary and a
planoconcave lens formed by the liquid secondary rainbow like concentric circles.
i.e., f = 45 cm
249. (c) Here, for yellow light r= 90° when i = C . As i is kept Applying Snell’s law,
sin i 1
same, C must be smaller for total internal reflection from = µ 2 = negative
1 sin r
µ= will be smaller, when µ is larger out of given
sin C sin i positive
⇒ sin r = 1 = = negative
colours, µ is largest for blue colour critical angle will be µ 2 negative
smallest for blue colour. Therefore, blue light would Since, sin r is negative, it means r must be on other side of
undergo total internal reflection. normal such that, total deviation of ray is, D = i + r.
250. (c) Here, µ =1. 5 255. (d) Here, an extended object lies immersed in water
If object lies on plane side contained in a plane trough. When seen from close to the
edge of the trough, the object looks distorted on an account
O of refraction of light from denser to rarer medium.
Therefore, apparent depths of the points close to the edge
and nearer to the surface of water is more compared to
(a) points away from the edge.
R1 = ∞ , R2 = − 20 cm Further, the angle subtended by the image of the object at the
eye is smaller than the actual angle subtended by the object in
1  1 1  1 1 1
= (µ − 1)  −  = (1. 5 − 1)  +  = air. Again,some of the points of the object far away from the
f  R1 R2   ∞ 20 40 edge may not be visible because of total internal reflection.
⇒ f = + 40 cm 256. (d) Here, in the adjacent figure, a pin is hold at L, mid-point
The lens behaves as convex. of AB. When seen from face AD (as long as i < C) image of
If object lies on its convex side R1 = 20cm, R2 = ∞. L appears to be at L′ closer to A.
L′ L
A B
O

i<C

(b) r
i >C
1  1 1  1 1 1 TIR
= (µ − 1)  −  = (1. 5 − 1)  −  = ⇒
f′  R1 R2   20 ∞  40 C D

f ′ = 40 cm
1 1
Thus, lens behaves as convex. From sin C = = = 0.625,sin −1 ( 0. 625 ) = 38.7
µ 1. 6
251. (b) The phenomenon involved in the reflection of
So, when angle of incidence becomes greater than
radiowaves by ionosphere is similar to total internal
C ( = 38.7° ) the rays starting from L will undergo total
reflection of light in as during a mirage (angle of incidence
internal reflection and pin shall not be seen at all.
> critical angle)
257. (a, b) A magnifying glass is used, as the object to be viewed
252. (b) In the given figure PQ is a ray of light passing through
can be brought closer to the eye than the normal near point.
focus and falling on the surface of a concave mirror. On
This results in a larger angle to be subtended by the object at
reflection from the mirror, the ray becomes parallel to
the eye and hence, viewed in greater detail. Moreover, the
principal axis of the mirror.
formation of a virtual erect and enlarged image, takes place.
253. (b) Ray number 2 follows the correct path according to the
258. (c) The least distance of distinct vision of an average person
laws of refraction.
(i.e., D) is 25 cm, in order to view an object with
Since, µ turpentine > µ water > µ air magnification 10.
So, as light travels from air to turpentine (i.e. from rarer to Here, v = D = 25 cm and u = f
denser medium), it bends towards the normal after refraction. D
Again when it strikes the turpentine water interface it must But the magnification m = v / u = D / f m =
f
bend away from the normal as it travels from denser to rare
D 25 1
medium. ⇒ f = = = 2.5 = 0.025 m ⇒ P = = 40 D
m 10 0.025
254. (a) For materials with negative refractive index, the
This is the required power of lens.
deviation produce after refraction is D = ( i + r ) i. e., light
bends such that, it is on other side of normal. 259. (a) Let the apparent depth be O1 for the object seen from m2 ,
For negative refractive index i.e., µ 2 is negative µ2 h
then O1 =
(refractive index of medium 2 w.r.t. medium 1 = 1µ 2 ) µ1 3
Since, apparent depth = real depth/refractive index, µ. 1 1 1
Using lens equation, − =
Since, the image formed by Medium 1, O2 act as an object v u f
for Medium 2.
uf ( − 50 ) × ( + 25 ) − 1250
If seen from µ 3 , the apparent depth is O2 . ⇒ v= = = = + 50 cm
u+ f − 50 + 25 − 25
Similarly, the image formed by Medium 2, O2 act as an
v + 50
object for Medium 3 Also, m = = = −1
u − 50
µ3 h  µ 3  h µ 2h h  µ 3 µ 2 
O2 =  + O1  =  +  =  + 

µ2 3  µ 2  3 µ 1 3 3  µ 2 µ 1  Thus, the image of A would have been formed at 50 cm
from the pole and 0.5 cm below principal axis.
If seen from outside, the apparent height is
So, coordinates of image ≡ ( + 50 cm, − 1 cm)
1 h  1 h h µ 3 µ 3 
O3 =  + O2  = +  +  262. (a) Let d be the diameter of the disc. The spot shall be
µ3  3  µ 3  3 3  µ 2 µ 1   invisible if the incident rays from the dot at O to the surface
at d / 2 at the critical angle.
h 1 1 1
=  + +  Let i be the angle of incidence.
3  µ1 µ 2 µ 3 
Using relationship between refractive index and critical angle,
This is the required expression of apparent depth. 1
then, sin t =
µ
260. (c) The relationship between refractive index, prism angle A Using geometry and trigonometry.
and angle of minimum deviation is given by d/2 d
Now, = tan i ⇒ = h tan i = h ( µ 2 − 1 )−1
 ( A + Dm ) h 2
sin  
µ=  2
Here, Dm = A 2h
∴ d=
 A
sin   µ2 −1
 2
This is the required expression of d.
sin A
∴ µ= 263. (a) (i) Let the power at the far point be P f for the normal
sin A / 2
relaxed eye of an average person. The required power
A A
2sin cos 1 1 1
On solving, we have = 2 2 = 2cos A Pf = = + = 60 D
A 2 f 0.1 0.02
sin
2 By the corrective lens the object distance at the far point is ∞.
A A The power required is
2sin cos
sin A 2 2 = 2cos A 1 1 1
∴ µ= = P f′ = = + = 50 D
sin
A
sin
A 2 f ′ ∞ 0.02
2 2 So for eye + lens system, we have the sum of the eye and
For the given value of refractive index, that of the glasses Pg
we have, ∴ P f′ = P f + Pg
A 3 A ∴ Pg = −10 D
∴ cos = or = 30°
2 2 2 (ii) His power of accommodation is 4 D for the normal eye.
∴ A = 60° Let the power of the normal eye for near vision be Pn .
This is the required value of prism angle. Then 4 = Pn − P f or Pn = 64 D
261. (a) Given, convex lens ( f = + 25 cm ). Since the lens is cut Let his near point be xn , then
along XX ′ (0.5 cm) above principal axis PP′, let us imagine 1
+
1
= 64 or
1
+ 50 = 64 ⇒
1
=14
the lens as if there is no cut. The point object placed at xn 0.02 xn xn
A ( − 50, 0 ) be treated as linear object AB. Also, u = − 50 cm. 1
∴ xn = = 0.07 m
Cut out along 14
Object (0, 0) this plane
(iii) With glasses Pn′ = P f′ + 4 = 54
A
5 cm 1 1 1 1
P B O P′ 54 = + = + 50 ⇒ =4
Principal axis xn′ 0.02 xn′ xn′
1
∴ xn′ = = 0.25 m
4

You might also like